cart-icon Товаров: 0 Сумма: 0 руб.
г. Нижний Тагил
ул. Карла Маркса, 44
8 (902) 500-55-04

Движение тела по наклонной плоскости решение задач: Решение задач на движение тел по наклонной плоскости. Видеоурок. Физика 11 Класс

Содержание

Движение тел по наклонной плоскости

«Требуются очень глубокие знания,

чтобы заметить простейшие, но

подлинные отношения вещей между собой».

Георг Кристоф Лихтенберг

Данная тема будет посвящена изучению движения тел по наклонной плоскости.

Задача 1. Тело массой 20 кг перемещается вверх по наклонной плоскости с углом наклона 45о и коэффициентом трения 0,03. Определите ускорение, с которым движется тело, если к нему параллельно основанию плоскости приложена сила 650 Н.

ДАНО:

РЕШЕНИЕ:

На основании второго закона Ньютона, составим уравнение движения тела

В проекциях на ось Оx:

В проекциях на ось

Оy:

Тогда сила трения равна

Следовательно

Преобразуем последнюю формулу

Тогда ускорение, с которым движется тело равно

 

Ответ: 14,8 м/с2.

Задача 2. Тело массой 4 кг перемещается вверх по наклонной плоскости под действием связанного с ним невесомой и нерастяжимой нитью грузом массой 12 кг. Начальные скорости тела и груза равны нулю, коэффициент трения тела о плоскость равен 0,05, угол наклона плоскости равен 30о. Определите ускорение, с которым движется тело, и силу натяжения нити. Считать, что блок невесом и трение в блоке отсутствует.

ДАНО:

РЕШЕНИЕ:

Запишем второй закон Ньютона для тела и груза

В проекциях на ось Ох:

В проекциях на ось Оy:

В проекциях на ось О’y’:

Получаем систему уравнений

Так как нить невесома и нерастяжима, а в блоке отсутствует трение, то:

С учётом последних выражений преобразуем систему уравнений

Для того, чтобы решить эту систему уравнений сложим первое и второе уравнение. Тогда получим

Преобразуем данное уравнение и выразим искомое ускорение

Теперь определим силу натяжения нити

Ответ: 6,4 м/с2; 43,2 Н.

Задача 3. Два груза массами т1 = 5 кг и т2 = 2 кг связаны невесомой нерастяжимой нитью, переброшенной через невесомый блок, который прикреплен к вершине призмы, и могут скользить по граням этой призмы. Определите ускорение грузов, если начальные скорости грузов равны нулю, α = 60о, β = 30о, а коэффициент трения — 0,3.

ДАНО:

РЕШЕНИЕ:

Запишем второй закон Ньютона для обоих грузов

В проекциях на ось

Оx и O’x’:

В проекциях на ось Оy и О’у’:

С учётом последних выражений преобразуем систему уравнений в проекциях на ось Оx и O’x’:

Так как нить и блок невесомы:

Так как нить нерастяжима и в блоке нет сил трения:

С учётом последних равенств сложим первое и второе уравнение системы в проекциях на ось Оx и O’x’:

Тогда искомое ускорение равно

Ответ: 2,6 м/с2.

Конспект урока по физике для 9 класса по теме «Движение тела по наклонной плоскости»

Государственное бюджетное общеобразовательное учреждение

лицей №144 Калининского района города Санкт-Петербурга

Конспект урока по физике

Тема: «Решение экспериментальных задач по теме

«Движение тела по наклонной плоскости»

Автор: Викулина Людмила Сергеевна

учитель физики ГБОУ лицея №144

г. Санкт-Петербург

2019 г

Предмет: Физика.

Тема: «Решение экспериментальных задач по теме «Движение тела по наклонной плоскости».

Класс: 9.

Тип урока: урок комплексного применения знаний.

Форма урока: урок-практикум по решению задач.

Цели урока:

Для учителя: обобщить теоретический материал и закрепить умение делать рисунок к задаче, находить проекции сил на оси; поддержать и увеличить интерес к теме.

Для ученика: обобщить свои знания по теме, уметь применять полученные знания на практике, развить умение работать в команде.

Задачи:

  • Развивающая: способствовать развитию внимания, логического мышления, умения анализировать исходные данные и полученные результаты, искать нестандартные подходы к решению задачи.

  • Обучающая: повторить изученный материал по теме «Движение тела по наклонной плоскости», научиться применять полученные знания на практике.

  • Воспитательная: развивать у учащихся навыки работы в команде, чувство ответственности за совместную деятельность, культуру общения.

Планируемые результаты

Предметные:

Умение анализировать задачу на движение тела по наклонной плоскости, создавать модель решения, выбирать необходимое лабораторное оборудование из предложенного перечня, планировать и выполнять измерения и вычисления, объяснять полученные результаты и делать выводы.

Метапредметные:

  • Познавательные: анализировать условия и требования задачи; уметь составлять рисунок к задаче с указанием всех сил, действующих на тело; уметь выбирать обобщенные стратегии решения, продолжить овладение экспериментальными методами исследования.

  • Регулятивные: выдвигать цели, планировать пути достижения цели, контроль своих действий, оценивать достигнутый результат.

  • Коммуникативные: самостоятельно организовывать учебное взаимодействие в группе, определять общие цели, распределять роли, договариваться друг с другом.

Личностные: уметь вести диалог на основе равноправных отношений и взаимного уважения; устойчивый познавательный интерес и становление смыслообразующей функции познавательного мотива.

Информационно-образовательная среда: компьютер; проектор; презентация, лабораторное оборудование: комплекты для каждой группы (наклонная плоскость, штатив, брусок, секундомер, динамометр), раздаточный материал на группу (карточки с заданиями, лист А3), листы самоконтроля, фломастеры.

Образовательная технология: интерактивный подход к обучению (работа в малых группах).

Формы работы на уроке: индивидуальная (решение теста), фронтальная (опрос), групповая (выполнение заданий).

Методы обучения: словесные (беседа, доклады учеников при защите решения задачи), наглядные (иллюстрации к заданиям теста, наблюдение при проведении эксперимента), лабораторный опыт.

Методы контроля и самоконтроля: фронтальный опрос, индивидуальный опрос, самоконтроль.

1. Организационный момент.

Включение учащихся в деятельность.

Приветствует учащихся; организует внимание учащихся и настрой на работу.

Задает вопрос: что мы узнали нового на прошлом уроке? Чему научились?

Учитель демонстрирует установку – брусок на наклонной плоскости.

Комментирует: сейчас этот брусок находится в состоянии покоя.

Что нужно сделать, чтобы он пришел в движение?

Медленно меняет угол наклона плоскости и задает вопрос: как узнать, на какой угол нужно приподнять эту плоскость? От чего это зависит?

Учитель: на прошлом уроке мы решали задачи количественные, когда нам из условия известны все необходимые величины для того чтобы найти нужную. Как вы думаете, какой тип задач мы будем решать сегодня?

Предлагает учащимся записать сформулированную ими тему урока.

Спрашивает, какую цель необходимо поставить на этом уроке и какими способами можно ее достичь.

Знакомит с листом самоконтроля (прил. 1).

Приветствуют учителя, включаются в урочную деятельность.

Отвечают, что научились применять алгоритм решения задач на второй закон Ньютона к ситуации, когда тело движется по наклонной плоскости.

Предлагают вариант: увеличить угол наклона плоскости.

Отвечают, что угол наклона, при котором тело начнет скользить, зависит от коэффициента трения между плоскостью и телом.

Догадываются о том, что речь идет об экспериментальных заданиях.

Записывают тему урока.

Формулируют цель и способы ее достижения.

Знакомятся с листом самоконтроля.

2. Актуализация знаний.

2.1. Повторение алгоритма решения задач.

Давайте повторим алгоритм решения задач по динамике на примере задачи.

Задача. Автомобиль массой 3 т движется вверх по склону горы (угол наклона дороги 30°). Чему должна быть равна сила тяги двигателя, чтобы автомобиль двигался равномерно? Коэффициент сопротивления движению 0,2.

Учащиеся по очереди выходят к доске, рассказывают шаг алгоритма и выполняют его.

Учащийся 1. Прочитать задачу, выделить данные величины, записать краткое условие задачи и определить, что нужно найти. Читает задачу, записывает краткое условие.

Учащийся 2. Сделать рисунок, на котором указать направления всех сил, действующих на тело. Выполняет рисунок.

Учащийся 3. Выбрать направление осей. Указывает на рисунке направление осей X и Y.

Учащийся 4. Определить, есть ли ускорение и указать его направление на рисунке. Комментирует, что автомобиль движется равномерно, значит, его ускорение равно нулю.

Учащийся 5. Записать второй закон Ньютона в векторном виде. Записывает первые два уравнения – второй закон Ньютона в общем виде, и равнодействующую силу.

Учащийся 6. Записать второй закон Ньютона в скалярном виде. Записывает второй закон Ньютона в проекциях на оси X и Y.

Учащийся 7. Решить полученную систему уравнений. Решает систему уравнения, приходит к выражению для силы тяги.

Учащийся 8. Выполнить вычисления и получить ответ. Проводит подстановку значений величин в итоговую формулу, получает ответ.

Учащийся 9. Проверить размерность величин. Проверяет правильность единиц измерений.

Учащийся 10. Оценить полученный результат. Делает вывод о том, что полученный ответ – 20 кН подходит для возможного значения силы тяги автомобиля.

Результат выполнения задания представлен в прил. 2.

2.2. Проверка теоретических знаний с помощью опроса в Plickers.

Давайте проверим, как вы усвоили материал прошлого урока. Приготовьте свои карточки.

Показывает на экране и озвучивает вопросы теста (прил. 3). Сканирует ответы учащихся мобильным приложением.

После опроса обсуждает с классом правильные ответы и ошибки учащихся.

Отвечают на вопросы теста, показывая свою карточку учителю.

Участвуют в обсуждении правильных ответов и анализируют свои ошибки.

Комментарий: Plickers – приложение, которое позволяет проводит фронтальные опросы с помощью одного мобильного телефона. Основу составляют мобильное приложение, сайт и распечатанные карточки с QR-кодами. Каждому учащемуся выдаётся по одной карточке, каждой стороне которой соответствует вариант ответа (A, B, C, D). Учитель с помощью мобильного приложения сканирует ответы детей в режиме реального времени [1]. В конце опроса учитель показывает на экране таблицу с результатами ответов учащихся. Каждый учащийся, зная номер своей карточки, может увидеть, на какие вопросы он ответил правильно, в каких ошибся.

3. Закрепление знаний и способов действий.

3.1. Работа в группах (решение экспериментальных задач).

Сейчас я предлагаю вам почувствовать себя настоящими учеными, исследователями, инженерами. Представьте, что ваша группа получила очередное задание (прил. 4) и на вас возложена важная миссия – справиться с ним успешно и быстрее своих конкурентов.

Напоминает о необходимости соблюдения правил работы с оборудованием. Контролирует работу класса.

Распределяют роли в группе. Анализируют задания, предлагают и обсуждают ход решения задачи. Выбирают необходимое оборудования, проводят измерения и выполняют вычисления, оценивают полученный результат.

Задания выстроены от простого к сложному. С первой задачей могут справиться все группы. Успешно справившись с первой задачей, учащимся легче решить вторую. Третья задача самая сложная, но даже у «слабой» группы есть возможность решить ее самым простым методом.

Группам, которые успешно справились с двумя заданиям, но третье выполнить не могут, учитель предлагает оформить на плакатах формата А3 решение предыдущих задач. В это время более «сильные» группы решают задание № 3, а первая справившаяся группа оформляет это решение.

Группа, решившая все задания раньше всех, приглашается в жюри.

3.2. Защита решения задач

Давайте сейчас проверим, как вы справились с возложенной на вас миссией и к каким результатам вы пришли.

Оценивать ваше решение будут ваши коллеги из группы № ___. Критерии оценивания: правильность решения, оформление, грамотность объяснения, ответы на вопросы. Давайте начнем с группы № ___.

Контролирует процесс разбора заданий. Задает вопросы учащимся. Делает акценты на главных моментах решения.

Команда, представляющая свое решение, выходит со своим плакатом, комментирует свое решение.

Другие команды внимательно слушают, сравнивают со своими результатами, после выступления задают вопросы, делают замечания.

Жюри оценивает выступление команды.

Основные моменты при разборе задач:

Задание № 1. Необходимо акцентировать внимание учащихся на том, что они должны подобрать такой угол наклона, при котором тело только начинает приходить в движение по наклонной плоскости.

Задание № 3. Учителю необходимо сделать акцент на то, что S в данном случае – расстояние, пройденное бруском вдоль наклонной плоскости, т.е. фактически часть длины наклонной плоскости L. Ряд учащихся может ошибиться при выполнении этого задания, посмотрев на рисунок задачи № 1 и неправильно измерить S. Другая возможная ошибка – учащиеся поняли, что расстояние измеряется вдоль наклонной плоскости, но неправильно измеряют расстояние с учетом габаритов бруска. Неправильные измерения приводят к числовому значению ускорения, значительно отличающегося от истинного.

Ускорения, вычисленные двумя способами, должны быть примерно равны.

Учитель задает вопрос: с чем связано то, что ускорения, полученные двумя способами, не совпадают идеально? Учащиеся приводят различные варианты, приводящие к небольшим отличиям в результатах.

4. Подведение итогов урока, выдача домашнего задания

Теперь, после такого тренинга, я уверена в том, что каждый сможет выполнить домашнее задание.

Задача. На наклонной плоскости длиной 5 м и высотой 3 м находится груз массой 50 кг. Какую силу, направленную вдоль плоскости, надо приложить, чтобы удержать этот груз? тянуть равномерно вверх? тянуть с ускорением 1 м/с2? Коэффициент трения 0,2.

Спрашивает: чем отличаются эти три случая? Какие силы действуют в каждом случае? Как движется тело в каждом случае?

Читают текст задачи.

Отвечают на вопросы учителя.

5. Рефлексия

Заполняют лист самоконтроля.

1. https://newtonew.com/app/provodim-opros-vsego-klassa-za-30-sekund-s-pomoshchju-plickers.html.

2. Рымкевич А.П. Физика. Задачник. 10-11 кл.: пособие для общеобразоват. учреждений. – М.: Дрофа, 2015 г.

3. Фастова Е.И., Иванова О.Л. Инновационные педагогические технологии. Кейс успешного педагога. — Волгоград: Учитель, 2017.

Приложение 1

Лист самоконтроля

ученика(цы) _______ класса ______________________________________________

Приложение 2

Образец выполнения задания

Приложение 3

Вопросы теста

Приложение 4

Карточка с заданиями группам

Задание группе № ____

Задание 1. Определите коэффициент трения покоя ластика о наклонную плоскость.

Задание 2. Брусок равномерно поднимают вверх по наклонной плоскости. Чему равен коэффициент трения скольжения?

Задание 3. Брусок скользит по наклонной плоскости. С каких ускорением он движется? Попробуйте измерить ускорение двумя способами: с точки зрения динамики и с точки зрения кинематики. Сравните полученные результаты. Данные для вычисления возьмите из задания № 2 (сила тяжести, коэффициент трения, sinα, cosα).

Приложение 5

Примеры выполнения экспериментальных заданий

Задание 1.

После решения задачи в общем виде учащиеся проводят необходимые измерения: высоту наклонной плоскости H, длину основания S. Затем вычисляют тангенс угла наклона, эта величина и будет равна коэффициенту трения покоя.

Задание № 2.

Учащиеся измеряют высоту наклонной плоскости h, длину наклонной плоскости L, силу тяги Fт, силу тяжести mg. Полученные данные подставляют в итоговую формулу, вычисляют коэффициент трения.

Задание № 3.

Способ № 1 (с помощью динамики).

У учащихся, решивших предыдущую задачу, есть все необходимые данные: sin и cos угла наклона, коэффициент трения. Поэтому после решения задачи в общем виде значение ускорения вычисляется быстро – достаточно просто подставить имеющиеся значений.

Способ № 2 (с помощью кинематики).

Движение бруска по наклонной плоскости будет равноускоренным без начальной скорости. Учащимся нужно просто вспомнить формулу перемещения и вывести из нее ускорение.

Учащиеся измеряют расстояние S, пройденное бруском, и время t, за которое это произошло.

Движение по наклонной плоскости | Презентация к уроку по физике (10 класс):

Слайд 1

Применение законов Ньютона 10 класс

Слайд 2

План решения задач по динамике 1. Сделать рисунок, на котором указать направление всех сил, приложенных к телу, направление ускорения, обозначить направление координатных осей . 2. З аписать в векторном виде уравнение второго закона Ньютона, перечислив в его правой части в любом порядке все силы, приложенные к телу 3. Записать уравнение второго закона Ньютона в проекциях на оси координат . 5. Найти численное значение неизвестной величины, если этого требует условие задачи . 4. Из полученного уравнения (системы уравнений) выразить неизвестную величину.

Слайд 3

Движение тел в горизонтальном направлении Какая горизонтальная сила потребуется, чтобы тело массой 2 кг, лежащее на горизонтальной поверхности, начало скользить по ней с ускорением 0,2 м/с 2 ? Коэффициент трения принять равным 0,02. Дано: m =2 кг μ = 0,02 а = 0,2 м/с 2 F — ? Решение: 1 3 4 2 5 Сделать рисунок В торой закон Ньютона в векторном виде Второй закон Ньютона в проекциях на оси координат решение в общем виде вычисление численного значения

Слайд 4

Движение тел в горизонтальном направлении Какая горизонтальная сила потребуется, чтобы тело массой 2 кг, лежащее на горизонтальной поверхности, начало скользить по ней с ускорением 0,2 м/с 2 ? Коэффициент трения принять равным 0,02. Дано: m =2 кг μ = 0,02 а = 0,2 м/с 2 F — ? Решение: 1 mg F тр N F а X ma = mg + F тр + N + F 3 О х : ma = — F тр + F (1) О у : 0 = — mg + N (2) из (2) : mg = N , т. к . F тр = μ N , получим уравнение ( 1 ) в виде: ma = — μ mg+ F Откуда F = ma + μ mg 4 2 5 Вычислим F = 0,79 Н Ответ: F = 0,79 Н у

Слайд 5

Движение тел по наклонной плоскости 

Слайд 6

h  l b tg  = h b cos  = b l , b = l· cos  sin  = h l , h = l· sin  Немного из тригонометрии…

Слайд 7

Движение тел по наклонной плоскости Движение без трения Трением между телом и поверхностью можно пренебречь Тело покоится на наклонной плоскости Сила трения покоя препятствует соскальзыванию тела Движение по наклонной плоскости с учетом силы трения Равноускоренное (равномерное) движение вниз или вверх

Слайд 8

 cos  = b l , b = l· cos  sin  = h l , h = l· sin  tg  = h b Тело покоится на наклонной плоскости (а = 0)

Слайд 9

 Тело покоится на наклонной плоскости (а = 0) Второй закон Ньютона в векторной форме ma = mg + N + F тр. пок В проекции на 0 = mgsin  — F тр О x : F тр = mgsin  0 = N — mgcos  О y : N = mgcos  F тр = μ N Как определить коэффициент трения μ ? С учетом

Слайд 10

h b Измерение коэффициента трения скольжения, используя наклонную плоскость Оборудование: линейка деревянная, брусок деревянный, линейка измерительная, штатив. tg  = μ tg  = h b μ = h b

Слайд 11

Движение по наклонной плоскости с учетом трения Задача. Какую силу надо приложить, чтобы равномерно поднять деревянный брусок по наклонной плоскости с углом наклона 30°, если известен коэффициент трения? 

Слайд 12

Какую силу надо приложить, чтобы равномерно поднять деревянный брусок массой m по наклонной плоскости с углом наклона 30°, если известен коэффициент трения? mg N F F тр .  У Х 0 a = 0 Дано: Решение: m  = 30° μ F — ? 0 = mg + F тр + N + F 1 2 3 О x : 0 = F – F тр – mgsin  (1) О y : 0 = N – mgcos  (2) 4 из (2): N = mgcos  , F тр = μ N = μ mgcos  F = μ mgcos  + mgsin  F = mg ( μ cos  + sin  ) Как проверить результат на опыте?

Слайд 13

Проверка результата на опыте Как установить наклонную плоскость с углом наклона 30°, если известна д лина основания линейки b = 0,5м ? Подсказки: h  l b tg 30° = h b , h = b · tg 30° h = 0,5м· √ 3 3 ≈ 0,23м h ≈ 0,23м b = 0,5м

Слайд 14

Домашнее задание Тело массой 2кг находится на наклонной плоскости. Коэффициент трения между телом и плоскостью 0,4. а)при каком угле наклона плоскости достигается наибольшее возможное значение силы трения? б) Чему равно наибольшее значение силы трения? в) Постройте примерный график зависимости силы трения от угла наклона плоскости.

Слайд 15

Задача Кирпич массой 2,5 кг лежит на доске длиной 2 м. Коэффициент трения между кирпичом и доской равен 0,4. а)на какую максимальную высоту можно поднять один конец доски, чтобы кирпич не сдвинулся? б) Чему будет равна при этом действующая на кирпич сила трения?

Условие неподвижности тела на наклонной плоскости. Методические материалы

Цифровой ресурс может использоваться для обучения в рамках программы основной и средней школы (базового уровня).

Компьютерная модель–задача. Используя предлагаемые начальные данные, пользователь должен ввести значение максимального угла наклона, при котором тело покоится на рассматриваемой наклонной плоскости.

Краткая теория

Брусок на наклонной плоскости покоится до определённого угла φ0, при превышении которого начинает соскальзывать вниз. Рассмотрим все силы, действующие на брусок при угле наклона α  < φ0.

Рис. 1. 

Так как брусок покоится, то векторная сумма всех сил должна быть равна нулю. Условия его равновесия имеют вид:

mg sin α = Fтр. пок.

После несложных преобразований получим, что, при α = φ0, tg φ0 = μпок.

Работа с моделью

Программой задается коэффициент трения. Пользователь имеет возможность ввести значение максимально возможного при этих условиях угла наклона, при котором тело еще покоится на наклонной плоскости. При нажатии на кнопку анализируется результат, после чего можно либо обновить экран для нового решения, либо посмотреть правильное решение данного задания.

Внимание! Модель допускает вариативность начальных условий. А при нажатии клавиши F5 значение коэффициента трения изменяется.

Рекомендации по применению модели

Данная модель может быть применена в качестве вспомогательного учебного средства на уроках решения задач в 10 классе по теме «Движение тела по наклонной плоскости».

Пример планирования урока с использованием модели

Тема «Практикум по решению задач на движение тела под действием нескольких сил. Движение тела по наклонной плоскости»

Цель урока: отработать решение задач на движение тала по наклонной плоскости.

№ п/п Этапы урока Время, мин Приемы и методы
1 Организационный момент 2
2 Объяснение нового материала по теме «Движение тела по наклонной плоскости» 15 Объяснение нового материала с использованием компьютерной модели «Устойчивость тела на наклонной плоскости»
3 Практикум по решению задач на движение тела под действием нескольких сил 25 Решение задач на доске с пояснениями, самостоятельное решение задач с использованием компьютерной модели «Устойчивость тела на наклонной плоскости»
4 Объяснение домашнего задания 3

Таблица 1.  

Движение тела по наклонной плоскости. Решение задач

Движение тела по
наклонной плоскости.
Решение задач 9-5.6.
Тело начинает соскальзывать с высоты Н по
наклонной плоскости с углом α. Начальная скорость
ϑо. Ускорение по наклонной плоскости а, скорость у
основания плоскости ϑ. После спуска тело проходит
по горизонтали до остановки путьS. Коэффициент
трения на наклонной плоскости и горизонтальном
участкеμ.
Тело начинает соскальзывать с высоты Н по
наклонной плоскости с углом α. Начальная скорость
ϑо. Ускорение по наклонной плоскости а, скорость у
основания плоскости ϑ. После спуска тело проходит
по горизонтали до остановки путьS. Коэффициент
трения на наклонной плоскости и горизонтальном
участкеμ.
N
α,о
Н,м
40
30
?
μ
3
0,15
S, м
?
?
27,54
Тело начинает соскальзывать с высоты Н по
наклонной плоскости с углом α. Начальная скорость
ϑо. Ускорение по наклонной плоскости а, скорость у
основания плоскости ϑ. После спуска тело проходит
по горизонтали до остановки путьS. Коэффициент
трения на наклонной плоскости и горизонтальном
участкеμ.
Решение:
Наклонная плоскость:
Н= L ∙sinα
m1a1= N1+ Fтр1+Fg1
ох: ma= Fg1∙sin α — μ∙N
oy: 0 = N — mg∙cosα
N = mg∙cosα
ma1= mg ∙sin α — μ∙mg∙cosα
a1= g∙sinα — μg∙cosα
Решение:
Горизонтальный участок пути: ma2= N+ Fтр+Fg
ох: ma2= — μ∙N
N = mg
ma2= — μ∙ mg
Н= L ∙sinα=4,9м
Домашняя работа:
Выполнить индивидуальный
вариант работы 9 – 5.6
N
α,о
Н,м
1
45
1,52
?
?
2
60
?
1
0,2
?
?
9,17
3
60
?
2
0,3
?
?
8,32
4
30
?
3
?
2,34
?
6,11
5
45
1,52
?
?
6,2
6
?
6
30
?
1
?
1,5
7
?
7
60
3,97
?
?
6,53
8
?
8
60
1,59
?
?
6. 53
5
?
9
60
?
1
0,3
?
?
6,11
10
45
?
2
?
4,16
?
4,58
11
30
?
1
?
2,34
9
?
12
30
2,20
?
0,15
?
6
?
13
30
3,09
?
0,15
?
?
16,65
14
45
1,53
?
0,2
?
?
6,37
15
30
2,41
?
0,15
?
6
?
16
45
?
3
0,25
?
?
7,33
17
30
?
1
?
3,63
?
16,65
18
30
2,15
?
0,25
?
?
5.09
19
30
4,04
?
0,25
?
?
9,98
20
60
?
3
?
7,26
?
7,33
21
45
?
2
?
5,89
?
16,65
22
60
1,34
?
0,15
?
5
?
23
30
2,87
?
?
2,34
?
6,11
24
45
3,05
?
0,25
?
?
9,98
25
45
1,26
?
0,15
?
?
8,49
μ
S, м
6,24
?
18,35

Материалы для организации дистанционного обучения.

Физика (7-9 классы)
Класс Название урока Ссылка на учебные материалы
7 Что изучает физика. Некоторые физические термины. Наблюдение и опыт https://resh.edu.ru/subject/lesson/2603/start/
7 Физика и техника

https://www.youtube.com/watch?v=Eta9kBhh03U 

7 Физические величины и их измерение. Измерение и точность измерения. Определение цены деления шкалы измерительного прибора. Определение объёма твёрдого тела https://resh.edu.ru/subject/lesson/2602/start/
7 Человек и окружающий его мир https://resh.edu.ru/subject/lesson/1526/start/
7 Строение вещества. Молекулы и атомы. Измерение размеров малых тел https://resh.edu.ru/subject/lesson/1533/start/
7 Броуновское движение. Диффузия. Взаимное притяжение и отталкивание молекул. Смачивание и капиллярность https://resh.edu.ru/subject/lesson/1534/start/
7 Агрегатные состояния вещества. Обобщение темы «Строение вещества» https://resh.edu.ru/subject/lesson/1532/start/
7 Механическое движение https://resh.edu.ru/subject/lesson/1488/start/
7 Виды механического движения. Равномерное и неравномерное движение

https://infourok.ru/videouroki/468 

https://infourok.ru/videouroki/421 

7 Скорость https://resh.edu.ru/subject/lesson/1525/start/
7 Инерция. Взаимодействие тел и масса. Измерение массы тела на уравновешенных рычажных весах https://resh.edu.ru/subject/lesson/1531/start/
7 Плотность и масса. Определение плотности твёрдого тела с помощью весов и измерительного цилиндра https://resh.edu.ru/subject/lesson/2601/start/
7 Решение задач по теме «Движение, взаимодействие, масса». Обобщение по теме «Движение, взаимодействие, масса» https://resh.edu.ru/subject/lesson/2974/start/
7 Сила. Сила тяжести https://resh.edu.ru/subject/lesson/2756/start/
7 Равнодействующая сила https://resh.edu.ru/subject/lesson/2973/start/
7 Сила упругости. Закон Гука. Динамометр. Градуировка динамометра. Исследование зависимости силы упругости от удлинения пружины. Определение коэффициента упругости пружины https://resh.edu. ru/subject/lesson/2600/start/
7 Вес тела. Невесомость https://resh.edu.ru/subject/lesson/2599/start/
7 Сила трения. Силы в природе и технике https://resh.edu.ru/subject/lesson/1536/start/
7 Решение задач по теме «Силы вокруг нас». Обобщение по теме «Силы вокруг нас» https://resh.edu.ru/subject/lesson/2972/start/
7 Давление. Способы увеличения и уменьшения давления. Определение давления эталона килограмма https://resh.edu.ru/subject/lesson/2971/start/
7 Природа давления газов и жидкостей. Давление в жидкости и газе. Закон Паскаля

https://resh.edu.ru/subject/lesson/2598/start/

https://mosobr.tv/release/7846

7 Расчёт давления жидкости на дно и стенки сосуда https://resh. edu.ru/subject/lesson/1537/start/
7 Сообщающиеся сосуды. Использование давления в технических устройствах https://resh.edu.ru/subject/lesson/1538/start/
7 «Решение задач по теме «Давление твёрдых тел, жидкостей и газов». Обобщение по теме «Давление твёрдых тел, жидкостей и газов» https://resh.edu.ru/subject/lesson/2970/start/
7 Вес воздуха. Атмосферное давление. Измерение атмосферного давления. Опыт Торричелли https://resh.edu.ru/subject/lesson/1535/start/
7 Приборы для измерения давления. Решение задач по теме «Атмосфера и атмосферное давление». Обобщение темы «Атмосфера и атмосферное давление» https://resh.edu.ru/subject/lesson/2969/start/
7 Действие жидкости и газа на погружённое в них тело https://resh.edu.ru/subject/lesson/2968/start/
7 Закон Архимеда. Плавание тел. Воздухоплавание https://resh.edu.ru/subject/lesson/2967/start/
7 Решение задач по теме «Закон Архимеда. Плавание тел». Обобщение по теме «Закон Архимеда. Плавание тел» https://resh.edu.ru/subject/lesson/2966/start/
7 Механическая работа. Мощность https://resh.edu.ru/subject/lesson/2965/start/
7 Энергия. Потенциальная и кинетическая энергия. Закон сохранения механической энергии. Изучение изменения потенциальной и кинетической энергий тела при движении тела по наклонной плоскости https://resh.edu.ru/subject/lesson/2597/start/
7 Источники энергии. Невозможность создания вечного двигателя. Решение задач по теме «Работа, мощность, энергия». Обобщение по теме «Работа, мощность, энергия» https://resh.edu.ru/subject/lesson/2964/start/
7 Простые механизмы https://mosobr. tv/release/7929
7 Рычаг и наклонная плоскость. Проверка условия равновесия рычага https://resh.edu.ru/subject/lesson/2963/start/
7 Блоки и система блоков. «Золотое правило» механики. Коэффициент полезного действия. Определение коэффициента полезного действия наклонной плоскости https://resh.edu.ru/subject/lesson/2962/start/
7 Решение задач по теме «Простые механизмы. «Золотое правило» механики. Обобщение по теме «Простые механизмы. «Золотое правило» механики» https://resh.edu.ru/subject/lesson/2596/start/
7 Виды механического движения. Равномерное и неравномерное движение https://infourok.ru/videouroki/468
7 Средняя скорость https://infourok.ru/videouroki/422
8 Температура и тепловое движение. Внутренняя энергия. Способы изменения внутренней энергии https://resh.edu.ru/subject/lesson/2595/start/
8 Теплопроводность. Конвекция. Излучение https://resh.edu.ru/subject/lesson/2594/start/
8 Количество теплоты. Удельная теплоёмкость. Расчёт количества теплоты https://resh.edu.ru/subject/lesson/2989/start/
8 Решение задач по теме «Внутренняя энергия». Обобщение по теме «Внутренняя энергия» https://resh.edu.ru/subject/lesson/2988/start/
8 Агрегатные состояния вещества https://resh.edu.ru/subject/lesson/2987/start/
8 Плавление и отвердевание кристаллических тел https://resh.edu.ru/subject/lesson/1539/start/
8 Удельная теплота плавления. Плавление аморфных тел https://resh.edu.ru/subject/lesson/2986/start/
8 Испарение и конденсация. Насыщенный пар. Кипение. Удельная теплота парообразования https://resh.edu.ru/subject/lesson/2985/start/
8 Влажность воздуха. Обобщение по теме «Изменения агрегатного состояния вещества» https://resh.edu.ru/subject/lesson/2984/start/
8 Энергия топлива. Принципы работы тепловых двигателей. Двигатель внутреннего сгорания. Паровая турбина. Реактивный двигатель. Холодильные машины. Тепловые машины и экология. Обобщение по теме «Тепловые двигатели» https://resh.edu.ru/subject/lesson/2593/start/
8 Электроскоп. Проводники и диэлектрики. Делимость электрического заряда. Электрон. Электризация тел. Электрический заряд https://resh.edu.ru/subject/lesson/2983/start/
8 Строение атомов. Ионы. Природа электризации тел. Закон сохранения заряда. Электрическое поле. Электрические явления в природе и технике https://resh. edu.ru/subject/lesson/1540/start/
8 Обобщение по теме «Электрическое поле» https://resh.edu.ru/subject/lesson/2592/start/
8 Электрический ток. Источники электрического тока. Гальванические элементы. Аккумуляторы. Электрический ток в различных средах. Примеры действия электрического тока https://resh.edu.ru/subject/lesson/2591/start/
8 Применение теплового действия электрического тока https://infourok.ru/videouroki/481
8 Электрическая цепь. Направление электрического тока. Сила тока https://resh.edu.ru/subject/lesson/2982/start/
8 Электрическое напряжение https://resh.edu.ru/subject/lesson/3126/start/
8 Электрическое сопротивление. Закон Ома https://resh.edu.ru/subject/lesson/2590/start/
8 Решение задач по теме «Электрический ток» https://resh. edu.ru/subject/lesson/2589/start/
8 Расчёт сопротивления проводника https://resh.edu.ru/subject/lesson/2980/start/
8 Последовательное и параллельное соединение проводников https://resh.edu.ru/subject/lesson/3246/start/
8 Сопротивление при последовательном и параллельном соединении проводников. Работа электрического тока. Закон Джоуля–Ленца https://resh.edu.ru/subject/lesson/2981/start/
8 Мощность электрического тока. Электрические нагревательные приборы https://resh.edu.ru/subject/lesson/2588/start/
8 Решение задач по теме «Расчёт характеристик электрических цепей». Обобщение по теме «Расчёт характеристик электрических цепей» https://resh.edu.ru/subject/lesson/2979/start/
8 Магнитное поле прямолинейного тока. Магнитное поле катушки с током https://resh.edu.ru/subject/lesson/2978/start/
8 Постоянные магниты. Действие магнитного поля на проводник с током. Электродвигатели. Магнитное поле Земли https://resh.edu.ru/subject/lesson/1541/start/
8 Электромагниты https://infourok.ru/videouroki/484 
8 Решение задач по теме «Магнитное поле». Обобщение темы «Магнитное поле» https://resh.edu.ru/subject/lesson/2587/start/
8 Система отсчёта. Перемещение. Перемещение и описание движения. Графическое представление прямолинейного равномерного движения https://resh.edu.ru/subject/lesson/3127/start/
8 Скорость при неравномерном движении. Ускорение и скорость при равнопеременном движении https://resh.edu.ru/subject/lesson/3128/start/
8 Перемещение при равнопеременном движении https://resh. edu.ru/subject/lesson/2977/start/
8 Решение задач по теме «Основы кинематики». Обобщение по теме «Основы кинематики» https://resh.edu.ru/subject/lesson/3129/start/
8 Инерция и первый закон Ньютона. Второй закон Ньютона https://resh.edu.ru/subject/lesson/2976/start/
8 Третий закон Ньютона. Импульс силы. Импульс тела. Закон сохранения импульса. Реактивное движение https://resh.edu.ru/subject/lesson/1542/start/
8 Решение задач по теме «Основы динамики». Обобщение по теме «Основы динамики» https://resh.edu.ru/subject/lesson/2975/start/
8 Итоговая проверочная работа https://resh.edu.ru/subject/lesson/3130/start/
8 Применение теплового действия электрического тока https://infourok.ru/videouroki/481
8 Электромагниты https://infourok. ru/videouroki/484
9 Относительность движения, сложение скоростей

https://infourok.ru/videouroki/336

https://infourok.ru/videouroki/560

9 Движение тела, брошенного вертикально вверх https://resh.edu.ru/subject/lesson/3025/start/
9 Движение тела, брошенного горизонтально https://resh.edu.ru/subject/lesson/3131/start/
9 Движение тела, брошенного под углом к горизонту https://resh.edu.ru/subject/lesson/3024/start/
9 Движение тела по окружности. Период и частота https://resh.edu.ru/subject/lesson/1530/start/
9 Закон всемирного тяготения https://resh.edu.ru/subject/lesson/2586/start/
9 Движение искусственных спутников Земли. Гравитация и Вселенная https://resh.edu.ru/subject/lesson/3022/start/
9 Решение задач по теме «Движение тел вблизи поверхности Земли и гравитация» https://resh.edu.ru/subject/lesson/3023/start/
9 Обобщение по теме «Движение тел вблизи поверхности Земли и гравитация» https://resh.edu.ru/subject/lesson/3021/start/
9 Механические колебания https://resh.edu.ru/subject/lesson/3020/start/
9 Маятник. Характеристики колебательного движения. Период колебаний математического маятника https://resh.edu.ru/subject/lesson/3019/start/
9 Гармонические колебания. Затухающие колебания. Вынужденные колебания. Резонанс https://resh.edu.ru/subject/lesson/3018/start/
9 Волновые явления. Длина волны. Скорость распространения волн https://resh. edu.ru/subject/lesson/3017/start/
9 Обобщение по теме «Электромагнитная природа света» https://resh.edu.ru/subject/lesson/3174/start/
9 Звуковые колебания и волны https://mosobr.tv/release/7951
9 Звуковые колебания. Источники звука https://resh.edu.ru/subject/lesson/2585/start/
9 Звуковые волны. Скорость звука https://resh.edu.ru/subject/lesson/3016/start/
9 Громкость звука. Высота и тембр звука https://resh.edu.ru/subject/lesson/3015/start/
9 Отражение звука. Эхо. Резонанс в акустике https://resh.edu.ru/subject/lesson/3014/start/
9 Ультразвук и инфразвук в природе и технике. Обобщение по теме «Звук» https://resh.edu.ru/subject/lesson/3013/start/
9 Индукция магнитного поля https://resh. edu.ru/subject/lesson/3012/start/
9 Однородное магнитное поле. Магнитный поток https://resh.edu.ru/subject/lesson/3132/start/
9 Электромагнитная индукция

https://resh.edu.ru/subject/lesson/3011/start/

https://mosobr.tv/release/7849

9 Переменный электрический ток https://resh.edu.ru/subject/lesson/3009/start/
9 Электромагнитное поле https://resh.edu.ru/subject/lesson/3010/start/
9 Электромагнитные колебания. Электромагнитные волны https://resh.edu.ru/subject/lesson/3008/start/
9 Механические и электромагнитные колебания https://mosobr.tv/release/7874
9 Механические и электромагнитные волны https://mosobr.tv/release/7885
9 Практическое применение электромагнетизма. Обобщение по теме «Электромагнитные колебания» https://resh.edu.ru/subject/lesson/2584/start/
9 Свет. Источники света https://resh.edu.ru/subject/lesson/3007/start/
9 Распространение света в однородной среде https://resh.edu.ru/subject/lesson/1543/start/
9 Отражение света. Плоское зеркало https://resh.edu.ru/subject/lesson/3006/start/
9 Преломление света https://resh.edu.ru/subject/lesson/3005/start/
9 Линзы https://resh.edu.ru/subject/lesson/3004/start/
9 Изображение, даваемое линзой https://resh.edu.ru/subject/lesson/3003/start/
9 Глаз как оптическая система. Оптические приборы. Обобщение по теме «Геометрическая оптика» https://resh.edu.ru/subject/lesson/3001/start/
9 Скорость света. Методы измерения скорости света https://resh.edu.ru/subject/lesson/3002/start/
9 Разложение белого света на цвета. Дисперсия света https://resh.edu.ru/subject/lesson/3000/start/
9 Механическое движение https://infourok.ru/videouroki/468
9 Качественные задачи по механике https://mosobr.tv/release/7941
9 Относительность движения, сложение скоростей https://infourok.ru/videouroki/336
9 Ускорение свободного падения на Земле и других планетах https://infourok.ru/videouroki/508
9 Свободное падение https://infourok.ru/videouroki/505
9 Первая космическая скорость https://infourok.ru/videouroki/308
9 Силы в природе https://uchebnik.mos.ru/moderator_materials/material_view/atomic_objects/1405905
9 Сила Ампера https://www.youtube.com/watch?v=ufLl9X5tgf0
9 Переменный электрический ток https://infourok.ru/videouroki/537
9 Экспериментальные методы исследования частиц https://www.youtube.com/watch?v=TKb79UHcVfA
9 Физико-математический практикум: экспериментальные задачи

 

https://mosobr.tv/release/7988

https://mosobr.tv/release/8016

«Физика в задачах и тестах»

Первый год обучения (68 часов)

1. Введение. Цели курса. ТБ при работе (2 часа)

2. Электронная таблица Excel, структура, панель инструментов (2 часа)

3. Этапы моделирования в Excel физических задач (4 часа)

4. Физическая задача. Классификация задач. Примеры задач всех видов. (С использованием моделирования) (4 часа)

5. Правила и приемы решения физических задач (6 часов)

6. Решение задач раздела «Механика» (48 часов)

6.1. Путь и перемещение. Основная задача механики. Векторы, действия с векторами. Сложение векторов. (4 часа)

6.2. Равномерное и равноускоренное движение. Равномерное движение. График скорости и перемещения. Движение с постоянным ускорением. Аналитический и графический метод решения задач. Равнозамедленное движение. Относительность движения. Относительность скорости. Относительность перемещения. Свободное падение. Падение тел при наличии начальной скорости. Движение тела, брошенного вверх. (10 часов)

6.3. Свободное падение тел. Свободное падение. Движение тела, брошенного вверх. Движение тела, брошенного под углом к горизонту. (4 часа)

6.4. Движение по окружности. Движение по окружности. Центростремительное ускорение. (4 часа)

6.5. Сила тяжести и вес тела. Сила тяжести. Вес. Перегрузки и невесомость. (4 часа)

6.6. Движение под действием нескольких сил. Алгоритм движения тела под действием нескольких сил. Движение по горизонтальной поверхности. Движение связанных тел. Движение по наклонной плоскости. (8 часов)

6.7. Статика. Задачи на определение характеристик равновесия физических систем. Рычаг. (4 часа)

6.8. Колебательное движение. Период и частота колебаний. Математический маятник. Пружинный маятник. (4 часа)

6.9. Упругие и неупругие соударения. Соударения упругих шаров. (4 часа)

6.10. Закон сохранения импульса. Реактивное движение. Закон сохранения импульса. Реактивное движение. Примеры реактивного движения в природе и технике. (2 часа)

6.11. Итоговое занятие. Конференция. Защита проектов. (2 часа)

 

Второй год обучения (68 часов)

1. Тепловые явления. Термодинамика. (26 часов)

1.1. Решение задач на расчет количества теплоты, требуемого для нагревания тела. (2 часа)

1.2. Решение задач на расчет количества теплоты, выделяемого при сгорании топлива. (2 часа)

1.3. Решение комбинированных задач. (Решение задач на расчет количества теплоты, требуемого для нагревания тела с учетом КПД). (Решение задач на расчет количества теплоты, выделяемого при сгорании топлива с использованием закона сохранения энергии). (6 часов)

1.4. Решение задач на расчет количества теплоты, необходимого для плавления или конденсации. (2 часа)

1.5. Решение комбинированных задач на первый закон термодинамики. Решение задач на тепловые двигатели. (6 часов)

1.6. Способы увеличения эффективности использования тепловых двигателей. Использование моделей (4 часа)

1.7. Решение конструкторских задач и задач на проекты: модель газового термометра, модель предохранительного клапана на определение давления, проекты использования газовых процессов для подачи сигналов, модель тепловой машины, проекты практического определения радиуса тонких капилляров. Использование моделей ИКТ. 4

2. Электростатика. (12 часов)

2.1. Решение задач на закон Кулона и закон сохранения заряда. Взаимодействие заряда в любой среде. (4 часа)

2.2. Нахождение потенциала и разности потенциалов (2 часа)

2.3. Определение электроемкости конденсатора. Соединения конденсаторов. Использование моделирования (6 часов)

3. Постоянный электрический ток (16 часов)

3.1. Решение задач на закон Ома для однородного проводника и смешанное соединение проводников. (4 часа)

3.2. Решение задач на закон Ома для замкнутой цепи. Нахождение ЭДС и внутреннего сопротивления источника тока. Решение задач на закон Ома для замкнутой цепи с смешанным соединением проводников (6 часов)

3.3. Использование амперметра и вольтметра для расширения предела измерения шкалы 4

3.4. Тепловое действие тока. Решение задач на закон Джоуля-Ленца. (2 часа)

4. Оптика. (12 часов)

4.1. Отражение света. Законы отражения. (4 часа)

4.2. Преломление света. (4 часа)

4.3. Линзы. Оптическая сила линзы. (4 часа)

5.Итоговое занятие. Конференция. Защита проектов (2 часа)

5.4 Наклонные плоскости — Физика

Задачи обучения разделу

К концу этого раздела вы сможете делать следующее:

  • Различия между трением покоя и кинетическим трением
  • Решение проблем с наклонными плоскостями

Поддержка учителей

Поддержка учителей

Цели обучения в этом разделе помогут вашим ученикам овладеть следующими стандартами:

  • (4) Научные концепции.Учащийся знает и применяет законы движения в двух измерениях в самых разных ситуациях. Ожидается, что студент:
    • (D) вычисляет влияние сил на объекты, включая закон инерции, взаимосвязь между силой и ускорением и характер пар сил между объектами.

Основные термины раздела

кинетическое трение статическое трение

Статическое трение и кинетическое трение

Вспомните из предыдущей главы, что трение — это сила, которая противодействует движению и постоянно находится вокруг нас.Трение позволяет нам двигаться, что вы обнаружили, если когда-либо пытались ходить по льду.

Существуют разные типы трения — кинетическое и статическое. Кинетическое трение действует на движущийся объект, а статическое трение действует на объект или систему в состоянии покоя. Максимальное статическое трение обычно больше кинетического трения между объектами.

Поддержка учителей

Поддержка учителей

[BL] [OL] Просмотрите понятие трения.

[AL] Начните обсуждение двух видов трения: статического и кинетического.Спросите студентов, какая из них, по их мнению, будет лучше для двух данных поверхностей. Объясните понятие коэффициента трения и то, что это число будет означать с практической точки зрения. Посмотрите на таблицу статического и кинетического трения и попросите учащихся угадать, какие другие системы будут иметь более высокие или более низкие коэффициенты.

Представьте, например, что вы пытаетесь сдвинуть тяжелый ящик по бетонному полу. Вы можете нажимать на ящик все сильнее и сильнее и вообще не двигать его. Это означает, что статическое трение реагирует на ваши действия — оно увеличивается, чтобы быть равным вашему толчку и в противоположном ему направлении.Но если, наконец, надавить достаточно сильно, кажется, что ящик внезапно соскользнет и начнет двигаться. Находясь в движении, легче удерживать его в движении, чем начать, потому что кинетическая сила трения меньше, чем сила статического трения. Если бы вам нужно было добавить массу к ящику (например, поместив на него коробку), вам нужно было бы толкать еще сильнее, чтобы он завелся, а также чтобы он продолжал двигаться. С другой стороны, если вы смазываете бетон маслом, вам будет легче запустить ящик и продолжать его работу.

На рис. 5.33 показано, как возникает трение на границе между двумя объектами. Увеличение этих поверхностей показывает, что они грубые на микроскопическом уровне. Поэтому, когда вы нажимаете, чтобы заставить объект двигаться (в данном случае ящик), вы должны поднимать объект до тех пор, пока он не сможет проскочить вместе с только кончиками поверхности, отломать точки или сделать и то, и другое. Чем сильнее прижимаются поверхности друг к другу (например, если на ящик ставится еще одна коробка), тем больше силы требуется для их перемещения.

Рисунок 5.33 Силы трения, такие как f , всегда противодействуют движению или попытке движения между соприкасающимися объектами. Трение возникает частично из-за шероховатости соприкасающихся поверхностей, как видно на увеличенном виде.

Величина силы трения имеет две формы: одна для статического трения, другая для кинетического трения. Когда между объектами нет движения, величина статического трения f s составляет

.

, где μsμs — коэффициент трения покоя, а N — величина нормальной силы.Напомним, что нормальная сила противостоит силе тяжести и действует перпендикулярно поверхности в этом примере, но не всегда.

Поскольку символ ≤≤ означает меньше или равно, это уравнение говорит, что статическое трение может иметь максимальное значение мксН.мксН. То есть

fs (max) = μsN.fs (max) = μsN.

Статическое трение — это сила реакции, которая увеличивается, чтобы быть равной и противоположной любой приложенной силе, вплоть до своего максимального предела. Как только приложенная сила превысит f с (макс.), Объект переместится.Когда объект движется, величина кинетического трения f k определяется как

, где μkμk — коэффициент кинетического трения.

Трение варьируется от поверхности к поверхности, потому что одни вещества более грубые, чем другие. В таблице 5.2 сравниваются значения статического и кинетического трения для разных поверхностей. Коэффициент трения зависит от двух соприкасающихся поверхностей.

Система Статическое трение мкс мкс Кинетическое трение μkμk
Резина на сухом бетоне 1.0 0,7
Резина на мокром бетоне 0,7 0,5
Дерево по дереву 0,5 0,3
Вощеное дерево по мокрому снегу 0,14 0,1
Металл по дереву 0.5 0,3
Сталь по стали (сухая) 0,6 0,3
Сталь по стали (промасленная) 0,05 0,03
Тефлон на стали 0,04 0,04
Кость смазана синовиальной жидкостью 0.016 0,015
Туфли по дереву 0,9 0,7
Обувь на льду 0,1 0,05
Лед на льду 0,1 0,03
Сталь на льду 0,4 0.02

Таблица 5.2 Коэффициенты статического и кинетического трения

Поскольку направление трения всегда противоположно направлению движения, трение проходит параллельно поверхности между объектами и перпендикулярно нормальной силе. Например, если ящик, который вы пытаетесь толкнуть (с силой, параллельной полу), имеет массу 100 кг, то нормальная сила будет равна его весу

. W = мг = (100 кг) (9,80 м / с2) = 980 Н, W = мг = (100 кг) (9.80м / с2) = 980 Н,

перпендикулярно полу. Если коэффициент трения покоя равен 0,45, вам придется приложить силу, параллельную полу, более

. fs (max) = μsN = (0,45) (980 N) = 440 Nfs (max) = μsN = (0,45) (980 N) = 440 N

для перемещения ящика. Когда есть движение, трение меньше, и коэффициент кинетического трения может быть 0,30, так что сила всего 290 Н

fk = μkN = (0.30) (980 N) = 290 Nfk = μkN = (0.30) (980 N) = 290 N

позволит ему двигаться с постоянной скоростью.Если бы пол был смазан, оба коэффициента были бы намного меньше, чем они были бы без смазки. Коэффициент трения является безразмерным и обычно представляет собой число от 0 до 1,0.

Работа с наклонными плоскостями

Ранее мы обсуждали, что когда объект опирается на горизонтальную поверхность, его поддерживает нормальная сила, величина которой равна его весу. До сих пор мы имели дело только с нормальной силой в одном измерении, с гравитацией и нормальной силой, действующей перпендикулярно поверхности в противоположных направлениях (сила тяжести направлена ​​вниз, а нормальная сила направлена ​​вверх).Теперь, когда у вас есть навыки работы с силами в двух измерениях, мы можем исследовать, что происходит с весом и нормальной силой на наклонной поверхности, такой как наклонная плоскость. Для задач с наклонной плоскостью легче разбить силы на составляющие, если мы повернем систему координат, как показано на рис. 5.34. Первый шаг при постановке задачи — разбить силу веса на компоненты.

Рисунок 5.34 На схеме показаны перпендикулярная и горизонтальная составляющие веса на наклонной плоскости.

Поддержка учителя

Поддержка учителя

[BL] Изучите концепции массы, веса, гравитации и нормальной силы.

[OL] Просмотрите векторы и компоненты векторов.

Когда объект лежит на наклоне, который составляет угол θθ с горизонталью, сила тяжести, действующая на объект, делится на две составляющие: силу, действующую перпендикулярно плоскости, w⊥w⊥, и силу, действующую параллельно плоскости. плоскость, w || w || . Перпендикулярная сила веса w⊥w⊥ обычно равна по величине и противоположна по направлению нормальной силе N.N. Сила, действующая параллельно плоскости, w || w ||, заставляет объект ускоряться вниз по склону. Сила трения ff противодействует движению объекта, поэтому он действует вверх по плоскости.

Важно соблюдать осторожность при разделении веса объекта на составляющие. Если угол наклона составляет угол θθ к горизонтали, то величины весовых составляющих равны

. w || = wsin (θ) = mgsin (θ) и w || = wsin (θ) = mgsin (θ) и w⊥ = wcos (θ) = mgcos (θ).w⊥ = wcos (θ) = mgcos (θ).

Вместо того, чтобы запоминать эти уравнения, полезно уметь определять их по разуму. Для этого нарисуйте прямоугольный треугольник, образованный тремя весовыми векторами. Обратите внимание, что угол наклона такой же, как угол между ww и w⊥w⊥. Зная это свойство, можно использовать тригонометрию для определения величины весовых составляющих

. cos (θ) = w⊥ww⊥ = wcos (θ) = mgcos (θ) cos (θ) = w⊥ww⊥ = wcos (θ) = mgcos (θ) sin (θ) = w || ww || = wsin (θ) = mgsin (θ).sin (θ) = w || ww || = wsin (θ) = mgsin (θ).

Поддержка учителя

Поддержка учителя

[BL] [OL] [AL] Поэкспериментируйте со скольжением различных предметов по наклонным плоскостям, чтобы понять статическое и кинетическое трение. Какие объекты должны скользить под большим углом? Что это говорит о коэффициентах трения этих систем? Требуется ли для начала движения объекта большая сила, чем для того, чтобы удерживать его в движении? Что это говорит о статическом и кинетическом трении? Когда объект скользит вниз с постоянной скоростью? Что это говорит о трении и нормальной силе?

Watch Physics

Компоненты силы наклонной плоскости

Это видео показывает, как вес объекта на наклонной плоскости разбивается на компоненты, перпендикулярные и параллельные поверхности плоскости.В нем более подробно объясняется геометрия для определения угла.

Проверка захвата

В этом видео показано, как вес объекта на наклонной плоскости разбивается на составляющие, перпендикулярные и параллельные поверхности плоскости. В нем более подробно объясняется геометрия для определения угла.

Когда поверхность плоская, можно сказать, что одна из составляющих гравитационной силы равна нулю; Который из? Когда угол наклона увеличивается, что происходит с величинами перпендикулярной и параллельной составляющих гравитационной силы?

  1. Когда угол равен нулю, параллельная составляющая равна нулю, а перпендикулярная составляющая максимальна.По мере увеличения угла параллельная составляющая уменьшается, а перпендикулярная составляющая увеличивается. Это связано с тем, что косинус угла уменьшается, а синус угла увеличивается.
  2. Когда угол равен нулю, параллельная составляющая равна нулю, а перпендикулярная составляющая максимальна. По мере увеличения угла параллельная составляющая уменьшается, а перпендикулярная составляющая увеличивается. Это потому, что косинус угла увеличивается, а синус угла уменьшается.
  3. Когда угол равен нулю, параллельная составляющая равна нулю, а перпендикулярная составляющая максимальна. По мере увеличения угла параллельная составляющая увеличивается, а перпендикулярная составляющая уменьшается. Это связано с тем, что косинус угла уменьшается, а синус угла увеличивается.
  4. Когда угол равен нулю, параллельная составляющая равна нулю, а перпендикулярная составляющая максимальна. По мере увеличения угла параллельная составляющая увеличивается, а перпендикулярная составляющая уменьшается.Это потому, что косинус угла увеличивается, а синус угла уменьшается.

Советы для успеха

Нормальная сила представлена ​​переменной N.N. Его не следует путать с символом ньютона, который также обозначается буквой N. Важно различать эти символы, тем более что единицы измерения нормальной силы (NN) оказываются ньютонами (N). Например, нормальная сила NN, которую пол прилагает к стулу, может быть N = 100 N.N = 100 Н. Одно важное отличие состоит в том, что нормальная сила — это вектор, а ньютон — это просто единица. Будьте осторожны, чтобы не перепутать эти буквы в своих расчетах!

Напомним, что процесс решения задач наклонной плоскости выглядит следующим образом:

  1. Нарисуйте схему проблемы.
  2. Определите известные и неизвестные количества и определите интересующую систему.
  3. Нарисуйте диаграмму свободного тела (которая представляет собой эскиз, показывающий все силы, действующие на объект) с системой координат, повернутой на тот же угол, что и наклонная плоскость.Разложите векторы на горизонтальные и вертикальные компоненты и нарисуйте их на диаграмме свободного тела.
  4. Запишите второй закон Ньютона в горизонтальном и вертикальном направлениях и сложите силы, действующие на объект. Если объект не ускоряется в определенном направлении (например, в направлении x ), тогда F net x = 0. Если объект действительно ускоряется в этом направлении, F net x = м а .
  5. Проверьте свой ответ.Разумный ответ? Единицы правильные?

Рабочий пример

Нахождение коэффициента кинетического трения на наклонной плоскости

Лыжник, изображенный на Рисунке 5.35 (a), с массой 62 кг скользит по снежному склону под углом 25 градусов. Найдите коэффициент кинетического трения лыжника, если известно, что трение составляет 45,0 Н.

Рисунок 5.35 Используйте диаграмму, чтобы найти коэффициент кинетического трения для лыжника.

Стратегия

Величина кинетического трения была задана как 45,0 Н. Кинетическое трение связано с нормальной силой Н как fk = μkNfk = μkN. Следовательно, мы можем найти коэффициент кинетического трения, сначала найдя нормальную силу лыжника на склоне. Нормальная сила всегда перпендикулярна поверхности, и поскольку нет движения перпендикулярно поверхности, нормальная сила должна равняться составляющей веса лыжника, перпендикулярной склону.

То есть

N = w⊥ = w cos (25∘) = mg cos (25∘). N = w⊥ = w cos (25∘) = mg cos (25∘).

Подставляя это в выражение для кинетического трения, получаем

fk = μkmg cos 25∘, fk = μkmg cos 25∘,

, которые теперь можно решить для коэффициента кинетического трения μ k .

Решение

Решение для μkμk дает

μk = fkw cos 25∘ = fkmg cos 25∘.μk = fkw cos 25∘ = fkmg cos 25∘.

Подставляя известные значения в правую часть уравнения,

μk = 45,0 Н (62 кг) (9,80 м / с2) (0,906) = 0,082. Μk = 45,0 Н (62 кг) (9,80 м / с2) (0,906) = 0,082.

Обсуждение

Этот результат немного меньше, чем коэффициент, указанный в Таблице 5.2 для вощеной древесины на снегу, но он все же разумен, поскольку значения коэффициентов трения могут сильно различаться.В подобных ситуациях, когда объект массой м скользит вниз по склону, составляющему угол θ с горизонтом, трение определяется как fk = μkmg cosθ.fk = μkmg cosθ.

Рабочий пример

Вес на уклоне, двумерная задача

Масса лыжника, включая снаряжение, 60,0 кг. (См. Рис. 5.36 (b).) (A) Каково ее ускорение, если трение незначительно? (б) Каково ее ускорение, если сила трения составляет 45,0 Н?

Рисунок 5.36 Теперь используйте диаграмму, чтобы найти ускорение лыжника, если трение незначительно и если сила трения составляет 45,0 Н.

Стратегия

Самая удобная система координат для движения на склоне — это та, в которой одна координата параллельна склону, а другая — перпендикулярна склону. Помните, что движения по перпендикулярным осям независимы. Мы используем символ ⊥⊥ для обозначения перпендикуляра и |||| означать параллель.

Единственными внешними силами, действующими на систему, являются вес лыжника, трение и нормальная сила лыжного склона, обозначенная на диаграмме свободного тела ww, ff и NN.NN всегда перпендикулярна откосу, а ff параллельна ему. Но ww не совпадает ни с одной из осей, поэтому мы должны разбить его на компоненты по выбранным осям. Определим w || w || быть составляющей веса, параллельной уклону, и w⊥w component составляющей веса, перпендикулярной уклону. Как только это будет сделано, мы можем рассмотреть две отдельные задачи: силы, параллельные склону, и силы, перпендикулярные склону.

Решение

Величина компонента груза, параллельного наклону, равна w || = wsin (25 °) = mgsin (25 °) w || = wsin (25 °) = mgsin (25 °), а величина Компонент груза, перпендикулярный уклону, равен w⊥ = wcos (25 °) = mgcos (25 °).w⊥ = wcos (25 °) = mgcos (25 °).

(a) Пренебрежение трением: поскольку ускорение параллельно наклону, нам нужно учитывать только силы, параллельные наклону. Усилия, перпендикулярные наклону, складываются в ноль, поскольку в этом направлении нет ускорения. Силы, параллельные склону, представляют собой вес лыжника, параллельный склону w || w || и трение фф. В предположении отсутствия трения по второму закону Ньютона ускорение, параллельное наклону, равно

. a || = Fnet || m, a || = Fnet || m,

Где результирующая сила, параллельная наклону Fnet || = w || = mgsin (25 °) Fnet || = w || = mgsin ( 25 °), так что

a || = Fnet || m = mgsin (25 °) m = gsin (25 °) = (9.80 м / с2) (0,423) = 4,14 м / с2a || = Fnet || m = mgsin (25 °) m = gsin (25 °) = (9,80 м / с2) (0,423) = 4,14 м / с2

является ускорение.

(b) Включая трение: теперь у нас есть заданное значение трения, и мы знаем, что его направление параллельно уклону и препятствует движению между контактирующими поверхностями. Таким образом, чистая внешняя сила теперь равна

. Fnet || = w || −f, Fnet || = w || −f,

и подставляя это во второй закон Ньютона, a || = Fnet || ma || = Fnet || m дает

а || = Fnet || m = w || −fm = mgsin (25 °) −fm.а || = Fnet || m = w || −fm = mgsin (25 °) −fm.

Подставляем известные значения, чтобы получить

a || = (60,0 кг) (9,80 м / с2) (0,423) -45,0 N60,0 кг, a || = (60,0 кг) (9,80 м / с2) (0,423) -45,0 N60,0 кг,

или

a || = 3,39 м / с2, a || = 3,39 м / с2,

, что соответствует ускорению, параллельному наклону, при наличии встречного трения 45 Н. \ circ \! от горизонтали.Какая составляющая силы веса параллельна наклону?

  1. 4.33 \, \ text {N}
  2. 5.0 \, \ text {N}
  3. 24,5 \, \ text {N}
  4. 42,43 \, \ text {N}

Snap Lab

Трение под углом: скольжение монеты

Объект будет скользить по наклонной плоскости с постоянной скоростью, если результирующая сила, действующая на объект, равна нулю. Мы можем использовать этот факт для измерения коэффициента кинетического трения между двумя объектами.Как показано в первом рабочем примере, кинетическое трение на склоне fk = μkmg cosθfk = μkmg cosθ, а составляющая веса на склоне равна mg sinθmg sinθ. Эти силы действуют в противоположных направлениях, поэтому, когда они имеют одинаковую величину, ускорение равно нулю. Выписывая эти

fk = Fgxμkmg cosθ = mg sinθ.fk = Fgxμkmg cosθ = mg sinθ.

Решая для μkμk, поскольку tanθ = sinθ / cosθtanθ = sinθ / cosθ, мы находим, что

μk = mg sinθmg cosθ = tanθ. μk = mg sinθmg cosθ = tanθ.

5.10

  • 1 монета
  • 1 книга
  • 1 транспортир
    1. Положите монету на книгу и наклоните ее до тех пор, пока монета не будет скользить по книге с постоянной скоростью. Возможно, вам придется слегка постучать по книге, чтобы монета сдвинулась с места.
    2. Измерьте угол наклона относительно горизонтали и найдите μkμk.

Проверка захвата

Верно или неверно — Если известны только углы двух векторов, мы можем найти угол их результирующего вектора сложения.

  1. Истинно
  2. Ложь

Проверьте свое понимание

17.

Что такое трение?

  1. Трение — это внутренняя сила, которая препятствует относительному движению объекта.
  2. Трение — это внутренняя сила, ускоряющая относительное движение объекта.
  3. Трение — это внешняя сила, препятствующая относительному движению объекта.
  4. Трение — это внешняя сила, увеличивающая скорость относительного движения объекта.
18.

Какие две разновидности трения? На что действует каждый?

  1. Кинетическое и статическое трение действуют на движущийся объект.
  2. Кинетическое трение действует на движущийся объект, а статическое трение действует на покоящийся объект.
  3. Кинетическое трение действует на покоящийся объект, а статическое трение действует на движущийся объект.
  4. Кинетическое и статическое трение действуют на покоящийся объект.
19.

Между статическим и кинетическим трением между двумя поверхностями, что имеет большее значение? Почему?

  1. Кинетическое трение имеет большее значение, потому что трение между двумя поверхностями больше, когда две поверхности находятся в относительном движении.
  2. Статическое трение имеет большее значение, потому что трение между двумя поверхностями больше, когда две поверхности находятся в относительном движении.
  3. Кинетическое трение имеет большее значение, потому что трение между двумя поверхностями меньше, когда две поверхности находятся в относительном движении.
  4. Статическое трение имеет большее значение, потому что трение между двумя поверхностями меньше, когда две поверхности находятся в относительном движении.

Поддержка учителей

Поддержка учителей

Используйте вопросы Check Your Understanding , чтобы оценить, достигли ли учащиеся учебных целей, поставленных в этом разделе. Если учащимся не удается решить конкретную задачу, «Проверьте свое понимание» поможет определить, какая цель вызывает проблему, и направит учащихся к соответствующему содержанию.

Силы, действующие на тело, движущееся по наклонной плоскости

Если пренебречь трением между телом и плоскостью, сила, необходимая для перемещения тела вверх по наклонной плоскости, может быть рассчитана как

F p = W h / l

= W sin α

= ma g sin α (1)

где

F p = тяговое усилие (Н, фунт f )

W = ma g

= сила тяжести — или вес тела (Н, фунты f )

h = высота (м, футы)

l = длина (м, футы)

α = угол места (градусы)

м = масса тела (кг, снаряды)

a г = ускорение свободного падения (9.81 м / с 2 , 32,174 фут / с 2 )

Путем добавления трения — (1) может быть изменен на

F p = W (sin α + μ cos α )

= ma g (sin α + μ cos α) (2)

где

μ = коэффициент трения

Пример — Сила тяги на наклонной плоскости

Корпус массой 1000 кг расположен на наклонной плоскости 10 градусов .Сила тяги без трения может быть рассчитана как

F p = (1000 кг) (9,81 м / с 2 ) sin (10 °)

= 1703 Н

= 1,7 кН

Онлайн-калькулятор силы в наклонной плоскости — единицы СИ

Калькулятор, представленный ниже, можно использовать для расчета тягового усилия, необходимого для перемещения тела вверх по наклонной плоскости.

Онлайн-калькулятор силы наклонной плоскости — Имперские единицы

Угол естественного откоса

Тело, покоящееся на плоскости, наклоненной под углом α к горизонтальной плоскости, находится в состоянии равновесия, когда сила тяжести имеет тенденцию скользить вниз по телу. наклонная плоскость уравновешивается равной и противоположной силой трения, действующей вверх по наклонной плоскости.

Для равновесия «угол отклика» α может быть выражен как:

μ = F p / F n = (W sin α) / (W cos α) = tan α (3)

Пример — Градиентная сила, действующая на автомобиль, выполненную работу и требуемую мощность

Вес Tesla Model X составляет 2400 кг . Сила, действующая на автомобиль с наклоном 5% , может быть рассчитана по формуле (1) как

F p_5% = (2400 кг) ( 9.81 м / с 2 ) sin (5 °)

= 2051 Н

Сила, действующая на автомобиль с наклоном 10% , может быть рассчитана как

F p_10% = (2400 кг) ( 9,81 м / с 2 ) sin (10 °)

= 4088 N

Если Tesla движется по наклонным дорогам с одинаковой скоростью — работа сделана силами после 1 км можно рассчитать как

Вт 5% = (2051 Н) (1000 м)

= 2051 кДж

Вт 10% = (4088 Н) (1000 м)

= 4088 кДж

Если скорость Tesla 80 км / ч (22.2 м / с) — время прохождения дистанции можно рассчитать как

t 80 = (1000 м) / (22,2 м / с)

= 45 с

Мощность, необходимая для перемещение транспортного средства (без качения и сопротивления воздуха) можно рассчитать как

P 5% = (2051 кДж) / (45 с)

= 45,6 кВт

P 50% = ( 4088 кДж) / (45 с)

= 90.8 кВт

Если скорость автомобиля снижается до 60 км / ч (16,6 м / с) — время прохождения дистанции можно рассчитать как

т 60 = (1000 м) / (16,6 м / с)

= 60,2 с

Мощность, необходимая для движения транспортного средства (без сопротивления качению и воздуха), может быть рассчитана как

P 5% = (2051 кДж) / (60,2 с)

= 34,1 кВт

P 50% = (4088 кДж) / (60.2 с)

= 67,9 кВт

Калькулятор наклонной плоскости

Этот калькулятор наклонной плоскости представляет собой инструмент, который помогает решать проблемы с наклонной плоскостью с учетом коэффициента трения. Читайте дальше, чтобы найти определение наклонной плоскости и распространенные примеры расчетной наклонной плоскости.

Что такое наклонная плоскость?

Наклонная плоскость может быть описана как плоская поверхность, которая поднимается с одной стороны так, что образует угол θ с землей.В повседневной жизни можно встретить примеры наклонных плоскостей, например, пандусы или дверные клинья. Фуникулер — это вид транспортного средства, в котором также используется концепция наклонной плоскости. . Идея простоты и полезности наклонной плоскости состоит в том, чтобы уменьшить силу , необходимую для подъема тела на некоторую высоту.

Основные параметры наклонной плоскости

Есть несколько характеристик, которые могут адекватно описать простую наклонную плоскость.Первичный — это наклон, связанный с уже упомянутым углом θ . Следующие — высота ( H ) — максимальный уровень над землей и — длина ( L ) — расстояние между вершиной и вершиной под углом θ . Вид сбоку наклонной плоскости можно представить в виде прямоугольного треугольника, поэтому при необходимости вы можете легко найти взаимосвязь между H , L и θ . Коэффициент трения — это еще одна особенность наклонной плоскости, которая указывает на наличие тормозной силы, которая влияет на движущееся тело или вообще не дает объекту двигаться.

Формулы наклонной плоскости для кубического блока

При решении задач такого типа всегда стоит найти силы, действующие на наш организм:

  1. Сила тяжести F г = м * г , где м — масса объекта, а г — гравитационная постоянная. Его можно разделить на две составляющие:

    • F i = F g * sinθ — параллельно наклонной плоскости
    • F n = F g * cosθ — перпендикулярно
  2. Сила трения, которая действует в противоположном направлении, как F i , но зависит от значения нормальной силы F n и коэффициента трения f : F f = f * F n

  3. Существует также сила реакции опоры Н с тем же значением, что и у F n и в противоположном направлении, но она не влияет на дальнейшие расчеты

Результирующая сила F вдоль наклонной плоскости может быть вычислена как разница между F i и F f и, таким образом, переписана как F = F i - F f = F g * (sinθ - f * cosθ)

Одно важное замечание: Вышеприведенное выражение чистой силы действительно только в том случае, если угол наклонной плоскости не превышает угол трения θ f , который можно оценить как tan (θ f ) = f .В противном случае сила трения компенсирует F i , и объект остается в покое.

С известным выражением результирующей силы несложно найти ускорение a , время скольжения t и конечную скорость V , используя формулы из калькулятора ускорения и значение начальной скорости V₀ :

  • a = Ф / м
  • t = (√ (V₀² + 2 * L * a) - V₀) / a
  • В = V₀ + a * t

Если объект начинает движение без начальной скорости, выражение для времени скольжения упрощается до:

Вращающиеся тела на наклонной плоскости

Нетрудно представить себе какой-нибудь круглый объект, который скорее скатится, чем , а не скользит , поэтому для вращающихся тел следует принять другой подход.На этот раз трение предотвращает скольжение предметов и одновременно позволяет вращение . Мы можем повторить процесс вычисления из предыдущего раздела, учитывая как поступательные, так и круговые движения, что довольно сложно, но, с другой стороны, мы можем использовать сохранения энергии . Он говорит нам, что сумма начальной потенциальной и кинетической энергии равна конечной кинетической энергии. Важно помнить, что кинетическая энергия вращения фиксируется в общей кинетической энергии.Формула ускорения изменяется следующим образом:

, где I — момент инерции объекта, а r — радиус между осью вращения и поверхностью наклонной плоскости, который обычно эквивалентен радиусу тела (например, шара или цилиндра). Остальные выражения для времени прокатки t и конечной скорости V точно такие же, как и ранее.

Кубический блок — несколько вычислительных примеров

  1. Предположим, что нам нужно найти время скольжения и конечную скорость скользящего объекта с этими входными данными: м = 2 кг , θ = 40 ° , f = 0.2 , H = 5 м , V₀ = 0 . Решение может быть получено с помощью следующих шагов:

    • рассчитать силу тяжести: F г = 2 кг * 9,807 м / с 2 = 19,614 Н
    • разделите его на две перпендикулярные составляющие: F i = 19,614 N * sin40 ° = 12,607 N , F n = 19,614 N * cos40 ° = 15,026 N
    • определить силу трения: F f = 0,2 * 15.026 N = 3,005 N
    • вычтите F i и F f , чтобы вычислить результирующую силу: F = 12,607 Н - 3,005 Н = 9,602 Н
    • , таким образом, можно получить ускорение: a = 9,602 Н / 2 кг = 4,801 м / с²
    • длина наклонной плоскости равна: L = 5 м / sin40 ° = 7,779 м
    • , поэтому можно получить время скольжения: t = √ (2 * 7,779 м / 4,801 м / с²) = 1,8 с
    • , а также конечная скорость: V = 4.801 м / с² * 1,8 с = 8,642 м / с

    Мы также можем оценить потери энергии, которые представляют собой разницу между начальной потенциальной энергией и конечной кинетической:

    • ΔE = м * г * H - м * V² / 2 = 2 кг * 9,807 м / с² * 5 м - 2 кг * (8,642 м / с) ² / 2 = 23,38 Дж .

    Полная энергия не сохраняется, это вызвано работой, совершаемой силой трения. Обычно он выделяется в виде тепла.

  2. Во втором примере найдем тот же параметр, но с разными значениями входных данных: м = 2 кг , θ = 20 ° , f = 0.5 , H = 5 м , V₀ = 0 . Во-первых, мы можем вычислить угол трения для данного коэффициента трения:

    • θ f = tan -1 (0,5) = 26,565 ° , что больше нашего угла θ .

    Это значит, что тело не двинется из-за достаточно большой силы трения! В результате нам даже не нужно повторять все эти шаги из предыдущего примера, потому что объект не может скользить вниз без какой-либо внешней силы.

  3. В последнем примере используются следующие данные: м = 2 кг , θ = 90 ° , f = 0 , H = 5 м , V₀ = 0 . На первый взгляд это может показаться странным, но давайте попробуем решить:

    • F г = 2 кг * 9,807 м / с 2 = 19,614 Н
    • F i = 19,614 Н * sin90 ° = 19,614 Н , F n = 19,614 Н * cos90 ° = 0 Н
    • F f = 0 N
    • F = F г = 19.614 N
    • a = 19,614 Н / 2 кг = 9,807 м / с² = g .

    Получается, что ускорение равно гравитационному. Угол θ = 90 ° обозначает вертикальное движение, а f = 0 указывает на отсутствие сопротивления, что означает, что мы сталкиваемся с проблемой свободного падения . Как только вы заметите это, вы можете найти время скольжения (падения) с помощью калькулятора свободного падения: t = 1.010 с .

Однако все эти результаты можно оценить без особых усилий, независимо от каких-либо дальнейших предположений — просто воспользуйтесь нашим калькулятором наклонной плоскости!

Катящийся шар

В итоге выясним время качения шара при начальных параметрах наклонной плоскости θ = 30 ° , H = 5 м и V₀ = 0 .Момент инерции твердого шара равен I = 2/5 * м * r² . Мы можем начать с расширения формулы для ускорения, помня, что результирующая сила равна F i = m * g * sinθ :

  • a = F i / (m + I / r 2 ) = m * g * sinθ / (m + (2 * m * r 2 ) / (5 * r 2 ) ) = 5 / 7 * g * sinθ = 3,502 м / с 2 .

Мы видим, что выражение для a значительно упрощается, и на самом деле существует общее правило, которое гласит, что для тел с моментом инерции в виде I = k * m * r² (где k — некоторый постоянный коэффициент) ускорение можно вычислить как:

Поразительно, что результат не зависит ни от массы, ни от размера мяча ! Остальное — известная процедура:

  • L = 5 м / sin 30 ° = 10 м
  • t = √ (2 * 10 м / 3.502 м / с²) = 2,390 с
  • V = 3,502 м / с² * 2,390 с = 8,369 м / с

ТРЕНИЕ М-2 И НАКЛОННАЯ ПЛОСКОСТЬ

Аппарат с наклонной плоскостью. [PASCO]
1. НАЗНАЧЕНИЕ

(1) Для измерения коэффициента трения для нескольких комбинаций материалов, делая использование наклонной плоскости.

(2) Изучить равновесие и неравновесие тела на наклонной плоскости под действие сил.

2. АППАРАТ

Рубанок наклонный, брусок, тележка на колесах, грузовой подвес, набор грузов, таймер, счетчик. В некоторых вариантах аппарата плоскости имеют деревянные плоскости, в некоторых — эмалированный металл.

3. ИСТОРИЯ ВОПРОСА

Обсуждение трения и проблемы движения по наклонной плоскости можно найти в любой учебник общей физики.Основное соотношение для трения:

и

, где μ s и μ k — константы пропорциональности, называемые соответственно коэффициентом статического трение и коэффициент кинетического трения.

В обычных обозначениях показаны силы, действующие на тело в наклонной плоскости. на рисунке 1. W — вес тела (W = мг), N — нормальная сила, возникающая из-за плоскости, f — сила трения, а P — приложенная сила, когда струна прикреплен к телу, чтобы подтянуть его вверх по плоскости.Обычно эта ситуация анализируется разделение сил на составляющие, параллельные и перпендикулярные плоскости, как показано на рис. 2. Составляющими груза являются:

    W = W x sin θ

    Вт = Вт y cos θ

Силы на блоке и диаграмма свободного тела.

4. ПРОЦЕДУРА

Примечание для студентов: в этом эксперименте будет взят большой объем данных.Чтобы помочь вам в организацию этих данных включены специальные таблицы данных.

(1) Пусковое трение по ровной плоскости . Положите деревянный брусок на плоскость с помощью его большим лицом вниз. Приложите к нему силу P с помощью струны, проходящей через шкив. на весовую вешалку. Добавляйте веса к вешалке, пока блок не начнет двигаться. В этом Способ определения примерного значения пускового трения. Определите ценность подробнее точно, начиная чуть ниже этого значения и добавляя вес очень маленькими приращениями.Погрешность можно оценить, отметив наименьший дополнительный вес, который дает движение.

Поместите 100 грамм на блок и повторите.

Поместите 200 грамм на блок и повторите.

НАПОМИНАНИЯ: Вы должны взвесить деревянный брусок. Струна всегда должна быть параллельна плоскости.

(2) Трение скольжения по горизонтальной плоскости . С весом на блоке, как и раньше, дайте блок толкнуть, чтобы преодолеть стартовое трение и снять грузы с подвески, пока блок скользит с почти постоянной скоростью.Будет довольно много неопределенности в определить, является ли скорость постоянной, поэтому будьте осторожны при этом. Определять внимательно оцените значение неопределенности P по этой причине.

В некоторых случаях может потребоваться очистка самолета с помощью моющего средства. Не трогайте рубанок руками, масло для тела оставит пятна.

(3) Равновесие на наклонной плоскости . Исследуйте как пусковое трение, так и трение скольжения под разными углами плоскости.Используйте углы 15 °, 25 °, 35 ° и 45 °. (Вы уже сделали это для 0 °.) Под каждым углом исследуйте ситуация с разными значениями W, включая следующие случаи: A) нет веса на блоке, B) 100 грамм на блок. C) 200 грамм на блок, и , если , то это раз , Г) 500 грамм на блок.

(4) Неравновесное состояние . Для этой части используйте длинный рубанок. Два плоских угла будут быть исследованным, 0 ° и 30 °.Определите силу P, необходимую для ускорения вверх. самолет со скоростью, удобной для измерения с помощью таймера. Измерьте время, чтобы разогнаться от покоя на измеренном расстоянии от плоскости. Самый простой способ сделать это — использовать расстояние на один метр: начните с нижней части подвески на высоте одного метра от пола. Отпустите вешалку, как только вы запустите таймер, и остановите таймер, как только вешалка коснется пол. Практикуйте операцию хронометража, пока не добьетесь воспроизводимых результатов.

Проделайте то же самое для ускорения по плоскости.

(5) Свободное тело на плоскости . Удалите шнур, приложивший силу, P. Используйте различные комбинации весов на теле для получения следующих экспериментальных ситуации.

а) блокировать статику на плоскости
б) блок скользит по плоскости с постоянной скоростью
в) блок ускоряется вниз по плоскости

(6) [По выбору инструктора] Проведите расследование, как в (1) — (3), используя малый четырехколесная тележка вместо блока.Используйте те же комбинации плоского угла и загрузка тележки, как предложено там. Трение намного меньше, чем с блоком; а также вы должны быть очень осторожны, чтобы уменьшить количество источников ошибок, чтобы получить хорошее измерение трение в колесах. Обратите внимание, что трение в колесах может быть сопоставимо с размер к трению в шкиве вверху плоскости. Некоторая изобретательность с вашей стороны будет требуется, чтобы разобраться в данных.

5.АНАЛИЗ

Части 1-4 эксперимента дают значения коэффициента трения для двух различных пары поверхностей: (1) дерево скользит по плоскости и (2) колеса катятся по плоскости самолет. Теория предсказывает, что каждый коэффициент трения должен быть постоянным. Однако ваши экспериментальные значения могут немного отличаться. Это вариация из-за случайной ошибки или остаточное несоответствие из-за систематической ошибки?

Чтобы ответить на этот вопрос, необходимо выполнить анализ ошибок.Вклад случайных Ошибка легко вычисляется по ошибке в ваших данных по стандартным формулам распространения ошибки. Используемое уравнение:

.
 

P - W sin θ μ = —————————— W cos θ

Ошибка в sin θ может быть получена из таблицы синусов. Например, если ваш угол был 25 ° ± 0,5 °, найдите в таблице значения sin 25 °; грех 24,5 ° и sin 25,5 °, чтобы получить диапазон неопределенности sin 25 °.Диапазон неопределенности будет разным для каждого угла, который вы использовали!

Основным источником ошибки в этом уравнении, вероятно, будет ошибка, связанная с взяв разность: P — W sin θ. Убедитесь в этом с помощью собственного анализа ошибок.

Сравните вашу рассчитанную вероятную ошибку в μ со средним отклонением вашего экспериментальные значения μ. Если среднее отклонение больше вероятной ошибки, вы вероятно, присутствует систематическая ошибка.

Проверьте изменение μ в ваших данных. Зависит ли μ от любого другого количества данных? Если так, который из? Попробуйте изобразить эту зависимость, указав вероятную ошибку в каждом значении μ с помощью флаг ошибки. Имеет ли эта зависимость какую-то простую форму? Можете ли вы предложить какую-либо физическую причину такая зависимость?

6. ВОПРОСЫ

(1) Будет ли коэффициент трения трехколесного автомобиля меньше, чем у четырехколесного? автомобиль, если предположить, что автомобили одинаковы по весу и колеса одинаковые? Рассчитать соотношение значений коэффициента трения в двух случаях.

(2) Рассмотрим шкив в верхней части самолета. Как происходит трение в шкиве зависит от натяжения струны и угла наклона плоскости? Выскажите свои предположения очевидно, и выводит уравнение трения шкива.

(3) Угол равномерного скольжения — это угол плоскости, при котором свободный блок скользить по плоскости с постоянной скоростью. Покажите, что этот угол равен

.

, где μ k — коэффициент кинетического трения.

Текст и рисунки © 1989, 2004 Дональд Э. Симанек.

Пример задачи о трении — Скольжение по наклонной плоскости

«Блок, скользящий по наклонной плоскости» — это обычная задача домашнего задания первого года, связанная с трением. Эта проблема представляет собой относительно простой пример проблемы, но ее можно сделать еще проще с помощью уловки. Хотя это не трюк с разрушением Земли, он не всегда приходит в голову начинающему студенту-физику. Это рабочее решение этой классической проблемы использует этот трюк.

Проблема:
Груз w стоит на ровной поверхности. Один конец поверхности поднимается до тех пор, пока блок не начнет скользить по рампе с постоянной скоростью. Какой коэффициент трения между блоком и наклонной плоскостью?

Решение:
На этом рисунке показаны силы, действующие, когда блок находится в движении.

Нормальная сила N перпендикулярна поверхности аппарели. Сила трения F f действует параллельно поверхности аппарели и препятствует движению блока.Груз w тянет вниз вертикально. Угол между землей и поверхностью пандуса равен θ.

А теперь самое интересное. Большинство учеников сразу же сделают свою систему координат параллельной земле (вверх-вниз, влево-вправо). Вы можете сэкономить много времени на тригонометрии, если решите выровнять систему координат с наклоном пандуса, а положительное направление оси x — вниз по пандусу. Видите ли, не Земля раскололась, как я сказал, но чрезвычайно полезно для упрощения проблемы. Теперь наши силы можно разбить на x- и y-компоненты.

В направлении x сумма сил равна:

ΣF x = w · sinθ — F f

Мы знаем силу трения F f = мкН. Поскольку блок движется, мы используем коэффициент кинетического трения: μ k .

ΣFx = w · sinθ — μ k N

Поскольку система находится в равновесии (постоянная скорость), сумма всех сил равна нулю.

w · sinθ — μ k N = 0
или
w · sinθ = μ k N

Теперь для направления y.Снова сумма сил равна нулю.

ΣF y = 0
ΣF y = N — w · cosθ

0 = N — w · cosθ
или
N = w · cosθ

Подставьте это решение в результат, полученный из x- направление.

w · sinθ = μ k (w · cosθ)

Решите относительно μ k

μ k = tanθ

Ответ:
Коэффициент кинетического трения между блоком и поверхностью наклонная плоскость равна касательной к углу, образованному между землей и поверхностью пандуса, когда блок движется с постоянной скоростью.

Для получения дополнительных сведений о силе трения, коэффициентах трения и другом примере проблемы перейдите по этой ссылке: Пример проблемы трения — Помощь в домашнем задании по физике.

Трение и наклоны — Уроки Wyzant

Автор — Алексей Ф.

Движение без трения в наклонных плоскостях

Движение по наклонной плоскости — это классическое применение второго закона Ньютона и диаграмм свободного тела. Вот типичная задача наклонной плоскости, которая игнорирует эффекты трения.В следующем разделе мы обсудим эффекты трения.

Пример 1: Блок массы m находится на склоне, составляющем угол θ с горизонтом. На блок не действует трение. Найдите

  1. Величина и направление действующей силы на блок в м, g и θ
  2. Величина и направление ускорения блока в единицах g и θ

Решение: На блок действуют силы тяжести F G и нормальная сила F N .Рисунок 1 иллюстрирует ситуацию со схемой свободного тела для блока. Обратите внимание, что пунктирные стрелки использовались для разделения F G на два компонента F G, ⊥ и F G, ∥, которые перпендикулярны и параллельны плоскости соответственно.

1: Чтобы определить направление чистой силы, F net , обратите внимание, что Второй закон Ньютона, F net = m a , подразумевает, что чистая сила имеет то же направление, что и ускорение. , а .Ускорение блока должно быть чисто вниз по плоскости, иначе блок упадет в самолет или спрыгнет с него, мы знаем, что этого не произойдет. Таким образом, F net направлен вниз по плоскости.

Для определения величины | F net | используем тот факт, что вектор F net является векторной суммой всех сил, действующих на блок

Поскольку F сеть расположена вдоль плоскости, все компоненты силы, перпендикулярные плоскости, должны в сумме равняться нулю.В этом случае у нас

Следовательно, у нас осталось

Таким образом, величина чистой силы, | F net | это просто величина | F G, ∥ |. Используя тригонометрию на треугольнике со штриховыми сторонами, мы имеем | F G, ∥ | = мгsin θ .

Таким образом, заключаем, что | F net | ​​= mgsin θ и что F net направлено вниз по плоскости.

2: Чтобы найти ускорение, примените Второй закон Ньютона в направлении F net , затем

Обратите внимание, что в этой задаче нормальная сила, F N , отменяется с помощью F G, ⊥, так что нам никогда не приходилось беспокоиться о вычислении величины F N . Это было удобно, но есть задачи на наклонной плоскости, в которых мы должны вычислить | F N | прежде чем мы сможем найти | F net | и .Лучшим примером этого является случай с самолетом с трением.

Трение

Трение — это сила, которая описывает способность соприкасающихся объектов сопротивляться относительному движению. Таким образом, сила трения на объекте всегда направлена ​​против направления движения объекта или предполагаемого движения. Мы скоро проясним значение термина «предполагаемое движение». Во-первых, нам нужно различать два типа сил трения: кинетическое трение и трение покоя.

Кинетическое трение

Кинетическое трение действует на объект, скользящий по другому объекту. Он всегда направлен против направления движения. Когда вы отправляете учебник скользящим по столу, сила кинетического трения о книгу — это то, что останавливает книгу.

Величина кинетической силы трения, ƒ k , на объекте

Где μ k называется кинетическим коэффициентом трения, а | F N | — величина нормального усилия поверхности на скользящий объект.Коэффициент кинетического трения полностью определяется материалами поверхностей скольжения. Например, древесина сосны, скользящая по оргстеклу, имеет фиксированное значение кинетического коэффициента трения. Важно помнить (и интересно размышлять) о том, что коэффициенты кинетического трения не зависят от площади поверхности контакта между скользящими объектами.

Статическое трение

Сила трения покоя, ƒ ​​ S , может быть охарактеризована следующим образом

  1. ƒ S действует против направления предполагаемого движения
  2. | ƒ S | ≤μ S | F N |

Свойство 2 можно выразить словами, поскольку величина силы статического трения всегда меньше или равна пороговому значению, заданному как μ S | F N |.Ниже мы обозначим это пороговое значение как ƒ с, = μ S | F N |. Подобно кинетическому трению, μ S называется коэффициентом статического трения и полностью определяется природой контактирующих материалов.

Пример должен прояснить оба эти свойства. Предположим, я кладу книгу на стол. Теперь я слегка толкаю книгу в направлении, параллельном столешнице. Если я толкну достаточно легко, книга не сдвинется с места. Таким образом, ускорение книги равно нулю, а из Первого (или Второго) закона Ньютона мы знаем, что силы, действующие на книгу, должны в сумме равняться нулю.Следовательно, в горизонтальном направлении должна быть какая-то другая сила, которая направлена ​​против моего толчка и имеет точно правильную величину, чтобы нейтрализовать эффект моего толчка. Эта сила и есть сила статического трения. Тот факт, что он действует противоположно направлению, в котором другие силы в совокупности ускоряют объект, объясняет утверждение свойства 1.

Мы можем продолжить этот пример, чтобы проиллюстрировать свойство 2. Предположим, я вдвое меньше силы толкаю книгу. Сила статического трения должна соответствующим образом регулировать свою величину; в противном случае возникла бы результирующая сила в горизонтальном направлении, противоположном направлению моего толчка.Книга действительно будет ускоряться в противоположном направлении, в котором я прилагаю силу, что абсурдно. Таким образом, fs предполагает любую величину, необходимую для предотвращения движения, вплоть до порогового значения ƒ с, max = μ S | F N |. Вот что означает свойство 2.

Неравенство в свойстве 2 алгебраически неудобно, и на практике обычно лучше использовать некоторую логику, чтобы не вводить неравенства в наши уравнения. Мы продемонстрируем это ниже.

В качестве последнего примечания по трению для любой данной пары поверхностей с коэффициентом статического трения μ S и кинетическим коэффициентом трения μ k верно, что μ S = μ S .Отличный способ понять это неравенство — это заметить, что если у нас есть блок, покоящийся на наклоне, угол которого относительно горизонтали мы медленно увеличиваем, тогда, как только блок поскользнется, он никогда сам по себе больше не остановится, даже если мы перестанем увеличивать угол. Это именно потому, что μ S = μ S .

Наклонная плоскость с трением

Пример 2: Деревянный блок массы m изначально находится в неподвижном состоянии на пластмассовой плоскости, наклоненной под углом 30 ° к горизонтали.Коэффициент статического трения для этой породы дерева и пластика составляет μ S = 1 / (2√3). Будет ли блок оставаться в покое в самолете?

Решение: Силы, действующие на блок, — это сила тяжести F G , нормальная сила F N и сила статического трения μ S . Рисунок 2 иллюстрирует ситуацию со схемой свободного тела для блока. Как и раньше, пунктирные стрелки разделяют F G на два компонента: F G, ⊥ и F G, ∥, которые перпендикулярны и параллельны плоскости, соответственно.

Поскольку блок изначально находится в состоянии покоя на плоскости, чтобы увидеть, скользит ли блок, мы должны сравнить величину μ S , необходимую для предотвращения скольжения (сделать a = 0 ), с пороговым значением μ S, макс. = μ S | F N |

Итак, предположим, что a = 0 , тогда Второй закон Ньютона дает

Поскольку μ S и F G, ∥ строго параллельны плоскости, а F G, ⊥ и F N строго перпендикулярны плоскости, эти две пары должны независимо суммироваться до нуля.Следовательно,

Где функции синуса и косинуса возникают из треугольника с пунктирными сторонами на рисунке 2.

Теперь мы должны интерпретировать эти два результата. Первое уравнение дает величину μ S , необходимую для уравновешивания той части силы тяжести, которая направлена ​​вниз по плоскости. Если μ S не может достичь необходимой величины, мг /2, блок выскользнет.

С другой стороны, мы можем объединить наши знания о | F <> <> | формула для μ S, max , чтобы получить

Таким образом, μ S, max = мг /4 меньше мг /2, величина силы статического трения, необходимая для предотвращения скольжения, поэтому блок скользит.

В качестве последнего примера рассмотрим следующее:

Пример 3: Предположим, что блок из последней задачи установлен скользящим по плоскости и μ K = 1 / 3√3. Какое ускорение блока?

Решение: ситуация проиллюстрирована на Рисунке 3.

Ускорение блока находится из Второго закона Ньютона

Так как a должно быть вдоль плоскости, все компоненты силы, перпендикулярные плоскости, в сумме равны нулю.В данном экземпляре

Уравнение принимает вид

Поскольку все силы в этом уравнении теперь действуют исключительно вдоль плоскости, мы можем описать различия в направлениях с помощью соглашения о знаках. Предположим, мы называем силы, направленные вниз по плоскости, положительными, тогда предыдущее уравнение эквивалентно

Следовательно,

Практические задачи

1. Блок помещается на плоскую доску, которую тащат по земле так, что он ускоряется со скоростью 3 м / с 2 .Блок остается неподвижным относительно доски. Какие силы действуют на блок в горизонтальном направлении

Ответ — A. Поскольку блок ускоряется вместе с доской в ​​горизонтальном направлении, в этом направлении действует результирующая сила. Источником горизонтальной силы является трение. Поскольку блок не скользит относительно доски, эта сила представляет собой статическое трение.

2. Предположим, что блок в предыдущем примере имеет массу 1 кг. Какое наименьшее возможное значение коэффициента трения покоя μ S , определяющее взаимодействие блока и платы? (предположим, что g = 9.8 м / с 2 )

Ответ — C. Поскольку мы ищем наименьшее возможное значение μ S , предположим, что сила статического трения достигает своего порогового значения, так что | ƒ s | = ƒ s, max = μ S | F N |. Поскольку статическое трение — единственная сила, действующая на блок в горизонтальном направлении, второй закон Ньютона дает

3. Предположим, что коэффициент статического трения между горизонтальной пластиковой поверхностью и прямоугольным деревянным блоком, лежащим на поверхности, равен μ S = 1/2, теперь металлический кирпич весом 2 кг приклеен к верхней части деревянного блока.Что из следующего верно относительно вероятности μ S и ƒ s, max , порога статического трения, между начальной и конечной ситуацией?

Ответ: B. μ S является характеристикой взаимодействия дерева и пластика и остается неизменной. С другой стороны, ƒ с, max = μ S | F N |. При добавлении металлического кирпича нормальная сила воздействия пластиковой поверхности на деревянный блок увеличивается. Отсюда ƒ с, увеличивается max .

4. Равномерный прямоугольный кирпич опирается на горизонтальную металлическую поверхность. К кирпичу прилагается сила в горизонтальном направлении, равная порогу силы статического трения. Теперь кирпич разрезают пополам по плоскости, параллельной металлическому листу. Две половинки размещаются встык на листе, и к одному концу прилагается одинаковая сила таким образом, что две половинки должны двигаться как единое целое. Какие из следующих утверждений верно?

Ответ — C. Система кирпича будет скользить только в том случае, если приложенная сила превысит пороговое значение статического трения ƒ с, макс. .Мы знаем, что в исходной ситуации приложенная сила точно уравновешивает ƒ с, максимум . Поскольку приложенная сила не меняется, нам просто нужно определить, как изменится ƒ с, максимум . Теперь ƒ с, max = μ с | FN | и μ s постоянна. Кроме того | FN | также постоянна, поскольку металлическая поверхность должна выдерживать один и тот же вес, хотя вес распределен по-разному. Таким образом, ƒ с, max без изменений.

Скользите вправо, используя наклонную плоскость — Урок

Быстрый просмотр

Уровень оценки: 4 (3-5)

Требуемое время: 45 минут

Зависимость урока: Нет

Тематические области: Геометрия, Физические науки, Решение проблем, Рассуждения и доказательства, Наука и технологии

Ожидаемые характеристики NGSS:


Резюме

Учащиеся изучают построение пирамиды, узнавая о простой машине, называемой наклонной плоскостью.Они также узнают о другой простой машине, шурупе, и о том, как ее использовать в качестве подъемного или крепежного устройства. Во время соответствующей практической деятельности учащиеся видят, как угол наклона и сила тяги могут облегчить (или усложнить) подтягивание объекта вверх по наклонной плоскости. Эта инженерная программа соответствует научным стандартам нового поколения (NGSS).

Инженерное соединение

Инженеры используют простые машины, чтобы строить более высокие, прочные и долговечные здания.В конструкции многих устройств используются наклонные плоскости. Например, пандусы представляют собой наклонные плоскости, которые упрощают перемещение людей в инвалидных колясках на более высокое место вместо использования лестницы. Парковочные гаражи спроектированы с использованием наклонных плоскостей для перевода автомобилей на следующий уровень парковки. Винты используются, чтобы поднять что-либо или скрепить две или более вещей вместе. Инженеры проектируют машины на основе больших винтов, такие как буровые установки для подъема масла или грязи, и используют винты меньшего размера для скрепления вместе самых разных предметов, от космического корабля до электрической зубной щетки.

Цели обучения

После этого урока учащиеся должны уметь:

  • Опишите наклонные плоскости и то, как инженеры используют их в повседневных задачах.
  • Определите винт как наклонную плоскость, обернутую вокруг стержня.
  • Объясните, как можно использовать винт для перемещения вверх материалов, например камня.
  • Опишите, как инженеры принимают во внимание простые машины при разработке современных проектов.

Образовательные стандарты

Каждый урок или задание TeachEngineering соотносится с одним или несколькими научными дисциплинами K-12, образовательные стандарты в области технологий, инженерии или математики (STEM).

Все 100000+ стандартов K-12 STEM, охватываемых TeachEngineering , собираются, обслуживаются и упаковываются сетью стандартов достижений (ASN) , проект D2L (www.achievementstandards.org).

В ASN стандарты иерархически структурированы: сначала по источникам; например , по штатам; внутри источника по типу; например , естественные науки или математика; внутри типа по подтипу, затем по классу, и т. д. .

NGSS: научные стандарты нового поколения — наука
Ожидаемые характеристики NGSS

3-ПС2-1.Спланируйте и проведите расследование, чтобы получить доказательства воздействия сбалансированных и неуравновешенных сил на движение объекта. (Класс 3)

Вы согласны с таким раскладом? Спасибо за ваш отзыв!

Нажмите, чтобы просмотреть другие учебные программы, соответствующие этим ожиданиям от результатов.
Этот урок посвящен следующим аспектам трехмерного обучения NGSS:
Наука и инженерная практика Основные дисциплинарные идеи Сквозные концепции
Совместно спланируйте и проведите расследование для получения данных, которые будут служить основой для доказательств, используя объективные тесты, в которых контролируются переменные и количество рассмотренных испытаний.

Соглашение о выравнивании: Спасибо за ваш отзыв!

В научных исследованиях используются различные методы, инструменты и техники.

Соглашение о выравнивании: Спасибо за ваш отзыв!

Каждая сила действует на один конкретный объект и имеет как силу, так и направление. На покоящийся объект обычно действует несколько сил, но они складываются, чтобы получить нулевую чистую силу на объект.Силы, которые не равны нулю, могут вызвать изменение скорости или направления движения объекта. (Граница: на этом уровне используется качественное и концептуальное, но не количественное сложение сил.)

Соглашение о согласовании: Спасибо за ваш отзыв!

Соприкасающиеся предметы оказывают друг на друга силу.

Соглашение о выравнивании: Спасибо за ваш отзыв!

Причинно-следственные связи обычно выявляются.

Соглашение о выравнивании: Спасибо за ваш отзыв!

Общие основные государственные стандарты — математика
  • Представляйте и решайте задачи, связанные с умножением и делением. (Оценка 3) Подробнее

    Посмотреть согласованную учебную программу

    Вы согласны с таким раскладом? Спасибо за ваш отзыв!

  • Умножайте или делите для решения словесных задач, связанных с мультипликативным сравнением, например.g., используя рисунки и уравнения с символом неизвестного числа для представления проблемы, отличая мультипликативное сравнение от аддитивного. (Оценка 4) Подробнее

    Посмотреть согласованную учебную программу

    Вы согласны с таким раскладом? Спасибо за ваш отзыв!

  • Применяйте и расширяйте предыдущие представления о делении, чтобы делить единичные дроби на целые числа и целые числа на единичные дроби.(Оценка 5) Подробнее

    Посмотреть согласованную учебную программу

    Вы согласны с таким раскладом? Спасибо за ваш отзыв!

Международная ассоциация преподавателей технологий и инженерии — Технология
ГОСТ
Колорадо — математика
  • Представляйте и решайте задачи, связанные с умножением и делением.(Оценка 3) Подробнее

    Посмотреть согласованную учебную программу

    Вы согласны с таким раскладом? Спасибо за ваш отзыв!

  • Умножайте или делите для решения словесных задач, связанных с мультипликативным сравнением.(Оценка 4) Подробнее

    Посмотреть согласованную учебную программу

    Вы согласны с таким раскладом? Спасибо за ваш отзыв!

  • Решайте реальные проблемы, связанные с делением единичных дробей на ненулевые целые числа и делением целых чисел на единичные дроби.(Оценка 5) Подробнее

    Посмотреть согласованную учебную программу

    Вы согласны с таким раскладом? Спасибо за ваш отзыв!

Предложите выравнивание, не указанное выше

Какое альтернативное выравнивание вы предлагаете для этого контента?

Больше подобной программы

Just Plane Simple

Этот урок знакомит студентов с тремя из шести простых машин, используемых многими инженерами.Эти машины включают наклонную плоскость, клин и винт.

Инженерное дело: простые машины

Студенты знакомятся с шестью типами простых машин — клином, колесом и осью, рычагом, наклонной плоскостью, винтом и шкивом — в контексте построения пирамиды, получая общее представление об инструментах, которые использовались с тех пор. древние времена и используются до сих пор.

Преимущества машин

На этом уроке учащиеся узнают о работе с точки зрения физики и видят, что работа упрощается благодаря использованию простых машин. Уже ежедневно сталкиваясь с простыми машинами, учащиеся узнают об их широко распространенном использовании для улучшения повседневной жизни.

Смотрите, как он скользит!

Учащиеся используют наклонные плоскости при воссоздании сложной задачи поднять каменный монолит для постройки пирамиды. Они сравнивают толкание и тягу блоков разного размера вверх по наклонной плоскости, определяют угол наклона и изучают изменения, которые происходят при увеличении или уменьшении угла…

Предварительные знания

Общие сведения о пирамидах. Знакомство с шестью простыми машинами, представленными в Уроке 1 этого раздела.

Введение / Мотивация

Простые машины помогают нам легче выполнять задачу.Какие из простых машин мы изучали? (Возможные ответы: клин, колесо и ось, рычаг, наклонная плоскость, винт и шкив.) Сегодня мы рассмотрим две из шести простых машин — наклонную плоскость и винт.

Наклонная плоскость часто является самой простой из всех простых машин , потому что она не перемещается, когда вы ее используете; он просто сидит на месте. Наклонная плоскость предназначена для перемещения чего-либо с более низкой высоты на более высокую. Наклонная плоскость может быть такой же простой, как подъездная дорожка или лестница.Вы когда-нибудь пользовались одним из них?

Наклонная плоскость помогает поднимать предметы на более высокий уровень. Вы когда-нибудь пробовали нести что-нибудь тяжелое по лестнице? Это довольно сложно! Как насчет того, чтобы поднять этот предмет по лестнице? Так проще? Это точно! Перенести тяжелый предмет по лестнице легче, чем по лестнице, а поднимать его по гладкому пандусу еще проще. Почему это так? (Ответ: Необязательно так много поднимать ногами.)

авторское право

Авторское право © Wikimedia Commons https: // commons.wikimedia.org/wiki/File:Approaching_the_Top_of_the_Ladder_to_Balcony_House,_Mesa_Verde_National_Park_(4851976994).jpg

Однако всегда есть компромисс, чтобы переместить что-то таким образом, чтобы потребовалось меньше усилий. В наклонной плоскости компромисс — расстояние. Если вы сравните длину лестницы с длиной пандуса, ведущего на второй этаж здания, вы обнаружите, что длина лестницы намного короче. Расстояние до пандуса больше, но для подъема требуется меньше усилий.Испытывали ли вы это механическое преимущество ? Люди из древних культур поняли это давным-давно, когда построили пирамид , используя длинные пандусы, чтобы помочь им перемещать тяжелые камни на вершину! Сегодня инженеры-механики используют наклонные плоскости во многих инженерных конструкциях для перемещения вверх, таких как гаражи, эвакуаторы, конвейерные ленты и эскалаторы.

A винт — другая форма наклонной плоскости; это просто наклонная плоскость, обернутая вокруг стержня, как спираль .(Если возможно, раздайте ученикам несколько винтов.) Винт также является второй простой машиной, которую мы собираемся изучить сегодня. Можете ли вы вспомнить повседневные примеры наклонных плоскостей или винтов? (Ответы: Лестница, пандус, горка, лестница, болт, винт, дрель.)

Хотя винт считается простой машиной, работа зависит от другой простой машины, рычага. Вы когда-нибудь видели или использовали винт, чтобы скрепить дерево? Как вкрутить шуруп в дерево? Ну, вы воспользуетесь отверткой или дрелью.Отвертка — это рычаг, который помогает вкрутить шуруп в дерево. Винт — это просто цилиндр, вокруг которого намотана наклонная плоскость. Заостренный конец винта работает как клин (еще одна простая машина!), Но винт отличается от клина, потому что он получает свою силу от поворота рычагом, а не от приложения прямой силы, чтобы толкнуть его в объект.

Винт может работать двумя способами: он может поднимать груз и может скреплять два или более объекта вместе.Пример использования винта для подъема груза — это когда он используется для заправки масла. Нефть, поступающая из глубокой скважины, легко откачивается с помощью винтового насоса. Архимед был известным математиком и изобретателем, который более двух тысяч лет назад сконструировал винт Архимеда — машину, которую поворачивали лошади или люди для подъема воды.

Когда мы используем шуруп для закрепления предметов, он преобразует вращательное движение при вращении шурупа в прямолинейное движение шурупа по дереву или другому материалу.Это то, что дает винту механическое преимущество . Чтобы превратить винт в твердый материал, требуется меньше усилий, чем для того, чтобы вбить клин в тот же материал. Сегодня инженеры используют винты во многих инженерных приложениях и конструкциях, таких как буровые установки, которые поднимают масло, грязь или воду. Вы когда-нибудь видели, чтобы домкрат поднимал машину, чтобы поменять спущенное колесо? Ну, это тоже пример винта. Инженеры также используют винты в качестве крепежа для больших объектов, таких как спортивные стадионы или самолеты, и для небольших объектов, таких как столы или MP3-плееры.Сегодня мы познакомимся поближе с двумя простыми машинами — наклонной плоскостью и винтовой. Как вы думаете, как они могли помочь в строительстве древних пирамид? После урока перейдите к упражнению «Смотри, как слайд»! для учащихся воссоздают сложную задачу поднять каменный монолит для построения пирамиды с использованием наклонных плоскостей.

Предпосылки и концепции урока для учителей

Механическое преимущество

Механическое преимущество машины — это отношение нагрузки к приложенной силе.Другими словами, механическое преимущество определяет, сколько силы нам нужно для выполнения задачи. Например, чем больше механическое преимущество машины, тем меньше силы требуется для выполнения такой задачи, как перемещение объекта. Верно и обратное. Математически механическое преимущество (MA) = нагрузка ÷ приложенная сила. Хорошим механическим преимуществом является то, что больше 1.

Плоскость наклонная

Назначение наклонной плоскости как простой машины — переместить что-либо с меньшей высоты на большую с меньшими усилиями.Объект, просто помещенный на наклонную поверхность, часто скользит по поверхности (см. Рисунок 1) из-за силы, действующей в направлении вниз. Другими словами, силы в этом сценарии неуравновешены (т.е. отсутствует сила, направленная вверх, чтобы противодействовать силе, направленной вниз, и, следовательно, объект будет скользить вниз). Скорость, с которой объект скользит вниз, зависит от того, насколько наклонена поверхность; чем больше наклон поверхности, тем быстрее предмет будет скользить по ней. Это измеряется углом наклона .Студенты могут найти это с помощью транспортира. Трение также влияет на движение объекта по склону. Трение — это сила, которая оказывает сопротивление движению, когда один объект соприкасается с другим. Теперь представьте, что вы находитесь с обратной стороны объекта и применяете силу, чтобы удерживать объект в одном и том же месте (не двигаясь). Чтобы удерживать объект в неподвижном состоянии, сила, которую вы должны приложить, должна была бы равняться направленной вниз силе силы тяжести. Это был бы пример сбалансированных сил.Если вы хотите подтолкнуть силу вверх, вам нужно будет превысить силу тяжести.

Рис. 1: На этой диаграмме показано, как древние культуры использовали наклонные плоскости для перемещения тяжелых камней на вершину своих пирамид. Сила тяжести, трения и тягового усилия — все это влияет на то, насколько легко (или сложно) вытащить тележку по наклонной плоскости. Авторское право

Авторские права © 2005 Трэвис Рейли, Программа ITL, Инженерный колледж, Университет Колорадо в Боулдере

Чтобы понять движение объекта по наклонной плоскости, важно проанализировать силы, действующие на него.Сила тяжести (также известная как вес) действует в направлении вниз. Когда угол наклона больше и круче наклон, приходится преодолевать больший весовой компонент. При меньшем уклоне весовая составляющая легче преодолевается и требует меньших усилий.

Механическое преимущество наклонной плоскости зависит от ее наклона и высоты. Чтобы найти идеальное механическое преимущество наклонной плоскости, разделите длину склона на его высоту.

Наклонная плоскость дает механическое преимущество, заключающееся в уменьшении силы, необходимой для перемещения объекта на определенную высоту; это также увеличивает расстояние, на которое должен перемещаться объект.Объект, движущийся вверх по наклонной плоскости, должен перемещаться по всей длине наклона плоскости, чтобы переместиться на расстояние высоты. Например, если у вас есть пандус с длиной уклона 20 метров, который поднимается на 5 метров в высоту, то ваш компромисс — это перемещение на расстояние 20 метров вместо подъема прямо на 5 метров, и ваше идеальное механическое преимущество составляет 4.

Винт

Винт — это простая машина, имеющая две цели. Его можно использовать для скрепления двух или более предметов вместе или для подъема тяжелого предмета.В большинстве случаев для поворота винта используется рычаг. Хороший пример — отвертка. Это окружность рычага или отвертки и шаг винта, которые определяют механическое преимущество винта.

Шаг винта — это расстояние между соседними резьбами на этом винте. Шаг можно рассчитать, разделив определенное расстояние на количество витков резьбы винта. Например, если у вас есть винтовая застежка с 5 витками резьбы на дюйм винта, то шаг винта будет 1/5.Один полный оборот винта в предмет равен расстоянию шага винта. Или, в этом примере, один поворот винта переместит винт на расстояние 1/5 дюйма.

Идеальное механическое преимущество винта находится приблизительно при делении окружности рычага на шаг винта. Например, если ключ на 8 дюймов используется для затяжки болта с шагом 1/5 дюйма, то идеальное механическое преимущество составляет π * 8 дюймов / (1/5 дюйма) = 126.

Если тот же самый винт с шагом 1/5 повернуть отверткой с диаметром окружности 1 дюйм, то идеальное механическое преимущество станет 1 ÷ 1/5 или 5.

Сопутствующие мероприятия

  • Смотрите слайд! — Учащиеся воссоздают сложную задачу поднять каменный монолит, чтобы построить пирамиду, используя наклонные плоскости. Они сравнивают толкание и тягу блоков или предметов разного размера вверх по наклонной плоскости. Они определяют угол наклона и изучают изменения, которые происходят при увеличении или уменьшении угла.

Закрытие урока

Сегодня мы узнали о двух простых машинах; наклонная плоскость и винт.Кто может мне привести пример наклонной плоскости? (Возможные ответы: пандус, лестница, эскалатор.) Как наклонная плоскость помогает нам в работе? (Возможный ответ: мы толкаем предметы вверх по наклонной плоскости.) Каков компромисс? (Ответ: Расстояние) Какими двумя способами используются винты? (Ответ: скреплять предметы или что-то поднимать.) Какая еще простая машина часто помогает нам использовать винт? (Ответ: рычаг.) Что сконструировал инженер, использующий наклонную плоскость или винт? (Возможные ответы: гараж, пандус, эскалатор, буровая установка, удерживание частей чего-либо вместе, например самолета или MP3-плеера.)

Проведите итоговую оценку, как описано в разделе «Оценка».

На других уроках этого раздела студенты изучают каждую простую машину более подробно и видят, как каждую из них можно использовать в качестве инструмента для построения пирамиды или современного здания.

Словарь / Определения

угол наклона: угол на дне пандуса, образованный пандусом и горизонтальной линией (см. рисунок 1).

трение: сила, препятствующая движению между двумя контактирующими телами.

наклонный: наклонный, наклонный или наклонный. Отклоняться от горизонтали.

наклонная плоскость: простая машина, поднимающая объект на большую высоту. Обычно это прямая наклонная поверхность и отсутствие движущихся частей, таких как пандус, наклонная дорога или лестница.

механическое преимущество: преимущество, полученное за счет использования простых машин, позволяющих выполнять работу с меньшими усилиями.Облегчение задачи (что означает меньшее усилие), но может потребоваться больше времени или места для работы (большее расстояние, веревка и т. Д.). Например, приложение меньшей силы на большем расстоянии для достижения того же эффекта, что и приложение большой силы на небольшом расстоянии. Отношение выходной силы, оказываемой машиной, к приложенной к ней входной силе.

Мезоамерика: регион, простирающийся на юг и восток от центральной Мексики и включающий части Гватемалы, Белиза, Гондураса и Никарагуа.В доколумбовые времена он был населен различными цивилизациями, такими как майя и ольмеки.

монолит: большой каменный блок, особенно используемый в архитектуре или скульптуре.

Шаг (винта): расстояние по вертикали между двумя витками резьбы винта.

сила тяги: причина движения или изменения.

пирамида: массивная структура древнего Египта и Мезоамерики, использовавшаяся для склепа или гробницы.Типичная форма — квадратное или прямоугольное основание на земле со сторонами (гранями) в форме четырех треугольников, которые встречаются в точке наверху. Мезоамериканские храмы имеют ступенчатые стороны и плоскую вершину, увенчанную камерами.

пандус: наклонная поверхность или проезжая часть, соединяющая разные уровни.

Винт: простая машина, которая поднимает или скрепляет материалы. Часто цилиндрический стержень, нарезанный спиральной резьбой.

простая машина: машина с небольшим количеством движущихся частей или без них, которая используется для облегчения работы (дает механическое преимущество).Например, клин, колесо и ось, рычаг, наклонная плоскость, винт или шкив.

уклон: наклонная линия, поверхность, плоскость, положение или направление. Величина или степень отклонения от горизонтали.

спираль: кривая, которая огибает фиксированную центральную точку (или ось) на постоянно увеличивающемся или уменьшающемся расстоянии от этой точки.

Оценка

Оценка перед уроком

Таблица «Знай / Хочу знать / Учиться» (KWL): Создайте классную диаграмму KWL, чтобы помочь организовать изучение новой темы.На большом листе бумаги или классной доске нарисуйте таблицу с заголовком «Простые машины: наклонные плоскости и винты». Нарисуйте три столбца с названиями K, W и L, представляющие, что учащиеся знают о наклонных плоскостях и винтах, что хотят, чтобы знал о наклонных плоскостях и винтах, и что они узнали о наклонных плоскостях и винтах и ​​их механических преимуществах . Заполняйте разделы K и W во время введения к уроку по мере появления фактов и вопросов.Заполните L-часть в конце урока.

Оценка после введения

Неформальное обсуждение: Запрашивайте, объединяйте и обобщайте ответы студентов на доске.

  • Какие бытовые примеры наклонных самолетов? (Возможные ответы: пандус для инвалидных колясок, тротуарный бордюр, который превращается в пандус для колясок и велосипедов, горка на детской площадке, горка, которая выбрасывает жевательные шарики из машины, лестница, эскалатор.)
  • Какие бывают примеры винтов? (Возможные ответы: буровая установка, автомобильный домкрат, болт, винтовые зажимы, которые скрепляют части чего-либо, например самолета или MP3-плеера.)
  • Как инженеры используют наклонные плоскости и винты в современных конструкциях? (Возможные ответы: наклонные плоскости включены в конструкцию конвейерных лент или пандусов, используемых для перемещения предметов вверх. Парковочные гаражи спроектированы с использованием больших наклонных плоскостей. Винты используются для скрепления объектов вместе, таких как самолеты и компьютеры, а также для подъема материала. , как при бурении на масло, воду или грязь.)

Итоги урока Оценка

Таблица KWL (Заключение): Как класс, завершите столбец L таблицы KWL, как описано в разделе «Оценка перед уроком».Составьте список всего, что студенты узнали о наклонных плоскостях и винтах, а также об их механических преимуществах. Были ли даны ответы на все вопросы W? Что нового они узнали?

Заключительное обсуждение: Попросите учащихся объяснить, почему легче вытащить тележку или заблокировать длинный неглубокий пандус, чем подниматься по ступеням, лестнице или крутому пандусу. Попросите их привести примеры механических преимуществ использования наклонной плоскости или винта.

Bingo: Раздайте каждому студенту лист бумаги со списком терминов лексики урока.Попросите каждого ученика обойти комнату и найти ученика, который сможет определить один словарный термин. Студенты должны найти разных студентов для каждого слова. Когда студент завершает все условия, он кричит «Бинго!». Продолжайте, пока двое или трое (или большинство) учеников не сыграют в лото. Спросите студентов, которые кричали «Бинго!» дать определения словарных терминов.

Использование уравнений: Предоставьте учащимся дополнительные примеры задач (аналогичные примерам, приведенным в предыстории урока), чтобы они сами вычислили механическое преимущество наклонной плоскости и винта.Попросите их использовать уравнения, приведенные в разделе «Предпосылки урока», для определения механического преимущества наклонной плоскости и винта.

  • Примеры: Пандус для инвалидов имеет длину 24 фута и поднимается на 2 фута в высоту. В чем механическое преимущество рампы? (Ответ: 24/2 = 12)
  • Стандартный винт ¼-20 имеет резьбу 20 / дюйм. В чем механическое преимущество использования отвертки с окружностью 2 дюйма? (Ответ: 2 / (1/20) = 40)

Исследование : Попросите учащихся самостоятельно исследовать сбалансированные и несбалансированные силы дома.Попросите их выполнить следующие действия и ответить на заданные вопросы:

  • Постройте две наклонные плоскости из повседневных материалов в вашем доме (например, ящиков для хлопьев, книг и т. Д.). Убедитесь, что у одного крутой склон, а у другого более пологий.
  • Выберите катящийся предмет (например, мрамор, игрушечную машинку и т. Д.).
  • Неуравновешенные силы: Поместите объект на вершину наклонной плоскости и наблюдайте, что он делает. Скатывается? Если да, то почему? (Ответ: объект должен скатиться в обеих плоскостях, потому что неуравновешенные силы вызывают движение объекта в направлении действующей силы.)
  • Уравновешенные силы: Теперь приложите палец к нижней части объекта, чтобы удерживать его на месте. Проделайте это с обоими самолетами. В каком самолете вам нужно было приложить больше силы, чтобы уравновесить силы? (Ответ: более крутая плоскость.) Когда силы уравновешены, движется ли объект? (Ответ: Нет. Уравновешенные силы заставляют объект оставаться на одном месте.)

Мероприятия по продлению урока

Предложите учащимся развить свое понимание наклонных плоскостей, обсуждая различные варианты использования пандусов.В какой ситуации вы можете сделать пандус короче / длиннее, мельче / круче? Когда можно добавить трение, чтобы пандус работал лучше? Нарисуйте разные пандусы для разных целей. Чем они отличаются?

Попросите учащихся взглянуть на ассортимент различных винтов и вычислить механическое преимущество каждого. Какой из них имеет наибольшее механическое преимущество и наименьшее? Попросите их вкрутить в дерево разные крепежи. Можете ли вы почувствовать и увидеть разницу в механическом преимуществе винта с близкой резьбой по сравнению с винтом.тот, у которого нити дальше друг от друга?

Попросите учащихся исследовать винт Архимеда и написать краткий отчет, описывающий, как работает это устройство, рисовать эскизы и приводить собственные примеры повседневного использования его для помощи людям.

Рекомендации

Хендерсон, Том. Урок 3: Силы в двух измерениях: наклонные плоскости. The Physics Classroom (учебник физики в средней школе). По состоянию на 25 января 2006 г.http://www.compadre.org/precollege/items/detail.cfm?ID=2014

Райт, Ричард. Дамы и господа … наклонный самолет! PCS Education Systems, Inc. По состоянию на 25 января 2006 г. http://www.weirdrichard.com/inclined.htm

Авторские права

© 2005 Регенты Университета Колорадо.

Авторы

Трэвис Рейли; Малинда Шефер Зарске; Лоуренс Э.Карлсон; Жаклин Ф. Салливан; Дениз В. Карлсон, при участии студентов, участвовавших в курсе подготовки инженерного корпуса К-12 весной 2005 года.

Программа поддержки

Комплексная программа преподавания и обучения, Инженерный колледж, Университет Колорадо в Боулдере

Благодарности

Содержание этих программ электронных библиотек было разработано в рамках Комплексной программы преподавания и обучения в рамках гранта GK-12 Национального научного фонда.0338326. Однако это содержание не обязательно отражает политику Национального научного фонда, и вам не следует предполагать, что оно одобрено федеральным правительством.

Последнее изменение: 30 апреля 2021 г.

.

Добавить комментарий

Ваш адрес email не будет опубликован. Обязательные поля помечены *